PDA

View Full Version : Consistent time travel, (ab)using the power



jseah
2011-11-23, 08:34 AM
I was contemplating what could be done with time travel and would like to ask the playground for ideas.

Specifically this scenario:
You gain a superpower that allows you to specify a point in time in the future or past within 1 week of the current time you are in now. You instantly travel to that time, as well as any small objects you are holding (+clothes) but not large suitcases or bags.

This is time travel of the consistent type (http://en.wikipedia.org/wiki/Novikov_self-consistency_principle). Aka, the most confusing to use. When you time travel backwards, there will exist two versions of you. Changing the past is not possible, because if you did change the past, it would not have been what it was before you traveled. (inconsistent time loops don't occur)

EDIT: from post #40:

<..> I'm not asking for answers to questions that arise from this. Nor am I even asking for the questions themselves.

I am just looking for more complex applications of the time travel power. More complex than those I just put up that is.

While I do acknowledge that many questions and problems are put up by my definitions of how this time travel superpower works, I am not looking for them.
I am aware of the implications on determinism and have thought about the sorts of experiments you can run with this. (one of them is to determine if there is a global hidden variable behind quantum mechanics. The Bell Test Experiments rule out local hidden variables, but not global ones)

There are two major assumptions with the existence of this time travel power.

1. The universe is deterministic, at least on the quantum level
2. In any time travel situation, paradoxes (inconsistent loops) never occur. A consistent time loop always exists regardless of the setup.

---------------------------------------------------------------

So, what can you do with this? I have thought of a few things:

Locking a key into its own box
You have a box and a key that locks it.
A future you arrives with key in hand and places the future key into the box. The present you locks the box with the present key then travels back into the past with the present key to be your future self.
Now, your 'future' self who is the only copy left is holding a locked box with the key to that lock inside the box.

Taking the key out of the box is similarly possible.

Time loop logic (http://en.wikipedia.org/wiki/Novikov_self-consistency_principle#Time_loop_logic)

The one-atom key
You have a locked box. You have lost the key and don't know where it is.
A future you arrives from the future with a key and unlocks the box.

In doing so, the key breaks and you repair it at a locksmith who reforges the metal into an identical key (to the eye at least) and finds he has to add a tiny amount.
The present you takes the reforged key and goes back into the past to be your future self.

The key in this loop is made of copies of the same few atoms (not enough by themselves to make a key) that the locksmith used to repair the key.

Future preemption
A random gunman on the run from the police holds you at gunpoint. Before he can pull the trigger, a future you appears with his gun and shoots him.
The present you then steals the gunman's gun and goes back in time to be the future you.

Future knowledge
You don't know you have this superpower. A future you appears and teaches you how to time travel so you can use it to go back into the past to teach yourself.

A more mundane version of this is obtaining future stock market prices 1 day ahead and using that to make insane amounts of money.

Or in the case of the one-atom key, your future self could find the key for you, give it to you and tell you where it was.

DoctorGlock
2011-11-23, 08:58 AM
Have you by any chance played continuum? Because this is making my PTSD from that flare up.

navar100
2011-11-23, 09:19 AM
http://www.imdb.com/title/tt0167720/

The Reverend
2011-11-23, 10:18 AM
Step one. Travel one week forward every day to read the news, observe stock prices, and copy lotto numbers.

Step two. Effectively become god.

Ravens_cry
2011-11-23, 10:38 AM
"Consistent" time travel gives me more headaches than a basket of weasels with tap dancing shoes inside a a reciprocating steam engine boiler.
Sure, when it only involves things like billiard ball collisions it can appear plausible, but when things get more complex, like with human beings, it gets patently absurd if we are allowed even a modicum of free will.

GungHo
2011-11-23, 10:51 AM
Step one. Travel one week forward every day to read the news, observe stock prices, and copy lotto numbers.
Avoid flying DeLoreans.

Blacky the Blackball
2011-11-23, 11:16 AM
Have you by any chance played continuum? Because this is making my PTSD from that flare up.

I (eventually) managed to get my hands on a copy of that game last week, and I'm still reading through it.

It seems to assume that the players and GM can think in four dimensions...

DoctorGlock
2011-11-23, 12:28 PM
I (eventually) managed to get my hands on a copy of that game last week, and I'm still reading through it.

It seems to assume that the players and GM can think in four dimensions...

Eh, understanding the time flow isn't that hard, you really just need to remember to keep causality intact. The annoying part is writing down every little detail because if you forgot anything you frag yourself into last century. And my GM remembered and wrote down everything. Still, it was good practice for keeping my DM on track when I played a time traveling wizard in 3.5.

Still... so much paperwork...

Also yeah, that book was impossible to get. And the few bits of narcissist floating around are even harder.

Zale
2011-11-23, 01:48 PM
This is why I hate time travel. :smalleek:

And in the last two, wouldn't you end up with an infinite number of yourselves having to go back forever?

Radar
2011-11-23, 03:01 PM
This is why I hate time travel. :smalleek:

And in the last two, wouldn't you end up with an infinite number of yourselves having to go back forever?
No, you loop only once.

Now if we are talking about fully consistant time travel, then you cannot possibly cause paradox, since every time travel is already accounted for (as in 12 Apes for example). You can use it to your advantage:

Precognitive harm immunity
Travel a week into the future and look for future you. If he's alive and unharmed, then nothing you'll do in the upcoming week will cause you significant harm. You can do any crazy stunt without winning Darwin Award!

jseah
2011-11-23, 03:08 PM
This is why I hate time travel. :smalleek:

And in the last two, wouldn't you end up with an infinite number of yourselves having to go back forever?
No.

In all the examples, the present you becomes the future you by going back in time.

By your own subjective experience of time, you lived through the same period of time twice.


Step one. Travel one week forward every day to read the news, observe stock prices, and copy lotto numbers.

Step two. Effectively become god.
This is just part of future knowledge.

You can do this too:
Observe stock market. Find a stock that has risen by an acceptable amount since yesterday.
Log into your account and "find" that you have 100 000 shares of that stock. Sell them all.
You time travel back in time to yesterday and buy those 100 000 shares. Time travel back since waiting is boring.

Now you just bought the shares you sold after you sold them. At least to you. To the stock market, you did it the normal way.

Note that this works with apples / physical objects as well.


"Consistent" time travel gives me more headaches than a basket of weasels with tap dancing shoes inside a a reciprocating steam engine boiler.
Sure, when it only involves things like billiard ball collisions it can appear plausible, but when things get more complex, like with human beings, it gets patently absurd if we are allowed even a modicum of free will.
Why not? Inconsistent loops don't happen. That's all.

Consistent loops do not rule out free will.

Free will + circumstance = action

Time loops are part of the circumstance portion. The only time loops that are consistent are those that, after you add free will, result in an action that you do time travel.

Free will not compromised.

Blacky the Blackball
2011-11-23, 03:44 PM
No, you loop only once.

Now if we are talking about fully consistant time travel, then you cannot possibly cause paradox, since every time travel is already accounted for (as in 12 Apes for example).

The way the Continuum game works, you can cause apparent paradox, but the conflict in your memories of what happened cause you "frag" until the paradox is resolved (usually by someone - possibly you - faking things so that the apparent paradox turns out not to have been one after all).


Precognitive harm immunity
Travel a week into the future and look for future you. If he's alive and unharmed, then nothing you'll do in the upcoming week will cause you significant harm. You can do any crazy stunt without winning Darwin Award!

Until your enemy kills you and then has someone impersonate you in order to fool your past self who travelled forward into thinking you'd still be alive when you actually wouldn't...

Radar
2011-11-23, 04:18 PM
Until your enemy kills you and then has someone impersonate you in order to fool your past self who travelled forward into thinking you'd still be alive when you actually wouldn't...
It's easy to counter: the best part of using secret passwords while contacting yourself is, that you never have to share it with anyone. If you contact your future self, you don't even need to have a set password - you can make one up right there and the future you should know it for obvious reasons.

Yes, the system isn't ideal, but it gives you a serious advantage.

kieza
2011-11-23, 04:21 PM
The bit I always liked about Continuum was the slipspan: need a screwdriver? It's in the first place you look. Need a getaway car? The first car you pass on the street has keys in the first place you look. You just have to go back sometime in the future when you have free time in order to set things up, and then return everything if you stole it from somewhere else. You can even wait a couple of months until errand day: go grocery shopping, fix leaky faucet, steal car for past self, mow lawn, pick up stolen car from past self and return to two seconds after you stole it, do dishes.

It even works for skills: if you need to know how to steal a getaway car, you just span to some timeframe where you're safe from interruption, spend a month practicing, and then span back to the moment when you needed to hotwire it.

It does require a little four-dimensional thinking, though. The one time I got a group to play it, they gave up around the time that they fragged themselves by accidentally killing someone before he could span back and frag them himself.

Fortuna
2011-11-23, 04:22 PM
The bit I always liked about Continuum was the slipspan: need a screwdriver? It's in the first place you look. Need a getaway car? The first car you pass on the street has keys in the first place you look. You just have to go back sometime in the future when you have free time in order to set things up, and then return everything if you stole it from somewhere else. You can even wait a couple of months until errand day: go grocery shopping, fix leaky faucet, steal car for past self, mow lawn, pick up stolen car from past self and return to two seconds after you stole it, do dishes.

It even works for skills: if you need to know how to steal a getaway car, you just span to some timeframe where you're safe from interruption, spend a month practicing, and then span back to the moment when you needed to hotwire it.

It does require a little four-dimensional thinking, though. The one time I got a group to play it, they gave up around the time that they fragged themselves by accidentally killing someone before he could span back and frag them himself.

Sounds like Bill and Ted: The Game. WANT!

Infernalbargain
2011-11-23, 04:28 PM
Well consistent time travel requires enforcing smooth temporal boundary conditions. With this in mind, none of the scenarios presented thus far are compatible with any kind of consistent time travel.

jseah
2011-11-23, 05:02 PM
Well consistent time travel requires enforcing smooth temporal boundary conditions. With this in mind, none of the scenarios presented thus far are compatible with any kind of consistent time travel.
Do you mind pointing out just how it breaks down?

What do you mean by smooth temporal boundary conditions?


Precognitive harm immunity
Travel a week into the future and look for future you. If he's alive and unharmed, then nothing you'll do in the upcoming week will cause you significant harm. You can do any crazy stunt without winning Darwin Award!
Well, you don't have to look, since you'll remember to be there.

The problem with this one is that by seeing your future self alive and healthy, does not garuantee that risky actions aren't risky.

All it does is say, "you are alive and healthy", and nothing about how you get there.

The trip you planned to go bungee jumping might be canceled. The plane you planned to skydive from has the pilot on break. etc.

Now, if your future self says "oh sure, the bungee jump was hilarious and awesome at the same time!", then you know that it will be safe.
Unless your future self was having a joke on you and you didn't really go after all. Or that it was terrifying and not at all fun. =P

You are trying to break into a military base with nothing but the clothes on your back. Mission is to steal the commander's bathroom keys. You don't know anything about the layout.

Future self appears and gives you a military grade laser pointer and a complete map of the base stitched from copies of various fire escape plans.

You get told to climb the electric fence. Which has been deactivated by a future self in the base. The alarm this would have caused is also deactivated by another future self. The guards watching the cameras are sleeping after a future you slipped them a shot of their own sleeping gas.

The landmine goes off in the distance, even if its currently in your hands. As the guards investigate, you climb over the fence. The guard in the watchtower isn't there as he failed to turn up for duty (legit, your future self knows this because he was told when he was you) and the person in charge of the log book can't find it (it turns up under the guy's bed later)

You follow the arrows on the map and communicate go/stop using the laser pointer with a future self camping out on the roof to avoid the guards investigating the mine.

Once inside, you deactivate the electric fence, alarm, wipe the logs and erase tracks. As an afterthought, the bathroom keys turn up in the commander's wine cupboard, forgotten under an old report, and you didn't need to search for it. A set of instructions on how to use the mine is acquired, and copies of every single fire escape plan.

Once outside, you bury the mine and set it off by trailing a heavy flat tire behind you, stolen from the garbage dump. Then you go steal the log book and return it a few hours later in the guy's bunk. You get reminded by the furthest future self over a phone to go use the sleeping gas.
In a mistake, you knock yourself out and a future you turns up with an 'acquired' gas mask to rescue you. A number of other future yous also exit the base around here.

After you wake up, you go back and tell yourself the required information in the right places.

----------------------------------------------------

Perhaps I should have clarified what I was looking for.

I am trying to think of a situation where you would need to have two interlocking loops.
I don't mean things like the hostage situation x8 (8 gunmen, 8 future yous to shoot them, etc.). That's just a loop nested in itself 8 times.

Eg. a future you and a past you appear. And somehow you need the past you to do something that the future yous can't.

hewhosaysfish
2011-11-23, 05:57 PM
Now, if your future self says "oh sure, the bungee jump was hilarious and awesome at the same time!", then you know that it will be safe.
Unless your future self was having a joke on you and you didn't really go after all. Or that it was terrifying and not at all fun. =P

Maybe he's only saying that to you because that's what he remembers being told when he was you and he doesn't want to "frag" himself...

jseah
2011-11-23, 06:18 PM
Maybe he's only saying that to you because that's what he remembers being told when he was you and he doesn't want to "frag" himself...
'fragging' yourself doesn't apply in this particular version of time travel.

This is consistent time travel. You won't be doing anything that doesn't match what happened / will happen / is happening.

IE. your future self won't be saying that because he wants the timeline to be consistent. If he wouldn't have said it, you wouldn't have heard it said.

Yukitsu
2011-11-23, 06:45 PM
The hardest thing I've ever done with consistent time travel was have my past self beat my future self without dying (in either incarnation) without meeting, and without paradox. The hardest part was that the character was a diviner.

The other thing I do, as watch closely for any sort of news of "X artifact was lost forever" through history. Because if it's going to be gone, who's to say I'm not the thief what did it? For my character, who had time travel going back significantly further, I had a pretty large backlog of old supposedly lost relics.

NichG
2011-11-23, 06:48 PM
The thing to remember is that consistent time travel is itself inconsistent with a use-at-will power. That is to say, someone could try to time travel with the express purpose of creating a paradox. The important thing to get down is what sort of things happen to prevent that, because for self-consistent time travel something must prevent it. To decide this, you need some sort of principle. I'm going to propose it to be the Path of Least Resistance.

So take Postcognitive Immunity for example. You've got someone who intends to go forward a week, find themselves alive, and then do all sorts of crazy stunts that would normally get them killed using the immunity given by self-consistent time travel. The easiest answer to this situation is that they find themselves dead every single time, of a massive brain hemorrhage that occurred instantly upon their return to the present. Or even better, they find themselves missing, and when they ask why, its because they traveled forward a week and never returned. Subsequently, for whatever reason, their power will not work to travel back into that week.

Path of Least Resistance generally rules out the spontaneous creation of time-looped objects like the one-atom key. The one-atom key requires that the key be in exactly the same configuration at two points in a loop. The more mass involved in this, the more unlikely that requirement is to be met only because of thermal agitation. So the chance of discovering a one-atom key equivalent looks like exp(-10^24). Even being a bit more lenient, you're far more likely to discover a one-atom key made of adamantine in a freezer than a one-atom key made of lead during summer, because of the reduced thermal effects and wear effects making it difficult to execute the loop.

So what can you still do under Path of Least Resistance? Non-loops are okay, like going back in time to save yourself. Ostensibly though the universe has to be paid off, because you dying is more likely (less opportunity for inconsistency) than you saving yourself and both are consistent outcomes. Paying off the universe would consist of somehow making it so that a self-consistent path is more likely in the long run if you survived than if you died, so it probably involves interacting with other time travelers. If, for example, you plan to kill another time traveler who travels more often than yourself, then the universe is more likely to let you save yourself.

You can also do the trick of locking the key inside a box that it opens, but your best bet is to do that trick with someone who isn't you. The reason being, if it is you you have to wait for yourself to show up, but if its someone else you can proactively seek them out. Better yet if its someone you never met until the travel event.

Re'ozul
2011-11-23, 06:53 PM
I like consistent time travel.

My favourite ways of using it:

When there is a big Lottery jackpot, stay away from your home city for the day before the Lottery Numbers are drawn as well as the day afterwards (avoid all interaction, cheap motel works, just sleep all day and night if you want or read a book).
Now drive home and see wether you claimed the price (did your future self leave you a message from the past?). If so, go back in time and claim the price avoiding yourself at the time you exist twice.

Day trading.
Rent a second flat, live two days for every one until you have enough money to stop. Though you will age twice as fast.

Yeah, standard get-rich stuff but I'd rather not temper with it too much.

Jothki
2011-11-23, 07:09 PM
Consistent time travel does have its occasional disadvantages. No Time to Explain (http://www.arcadebomb.com/play/no_time_to_explain.html) is a good example. It's not completely consistent, but close enough to show the point.

Infernalbargain
2011-11-23, 09:40 PM
Do you mind pointing out just how it breaks down?

What do you mean by smooth temporal boundary conditions?

Imagine a wave function that completely describes the state of everything in the entire universe. By smooth, I mean that there are no sudden changes in it (like there is in the step function) and there are no sudden changes in its rate of change (like there is in the absolute value function). An example of a temporal boundary condition is that at 4am CDT I was asleep; basically it is an association of a state with a time.

The best way to think of a time machine is as a probability warping device. This solves a lot of the famous paradoxes such as the grandfather paradox. Since the universal wave function can collapse into any particular state upon observation, we must ask what is the probability of it collapsing into a particular state? Which is more probable, your grandson appearing out of the time machine and killing you or your time machine breaks? It takes a lot of probability warping in order for a person to travel back in time (because the wave function needs to be perturbed greatly to produce another person), thereby a lot of normally immensely improbable things would become suddenly highly probable in the presence of any time machine powerful enough to transport a person. Suffices to say that we quite frankly cannot predict what will happen. At all. And no, whatever you're thinking of will not be certain.

Ravens_cry
2011-11-23, 10:26 PM
Why not? Inconsistent loops don't happen. That's all.

Consistent loops do not rule out free will.

Free will + circumstance = action

Time loops are part of the circumstance portion. The only time loops that are consistent are those that, after you add free will, result in an action that you do time travel.

Free will not compromised.
Let's say I go back in time and using period material I fashion an explosive device and I decide to set it off in a certain place at a certain time, because when I witnessed that time and place without time travel and there was no explosion.
Or even the simple task of meeting myself for the less homicidally inclined.
What's stopping me?
The "principle" says there is no paradoxes and there is only one timeline, yet it is so easy to create one in such a world as it could practically be said to be harder not to create paradoxes with time travel.

Blacky the Blackball
2011-11-24, 03:37 AM
'fragging' yourself doesn't apply in this particular version of time travel.

This is consistent time travel. You won't be doing anything that doesn't match what happened / will happen / is happening.

The way Continuum works (that's the game that uses the context of frag), time travel is consistent.

It is the apparent inconsistency that causes frag - and that apparent inconsistency must be resolved (by very careful actions by yourself or another time traveller) for the frag to go away.

Any true paradox that does not get resolved will end up destroying the universe.


IE. your future self won't be saying that because he wants the timeline to be consistent. If he wouldn't have said it, you wouldn't have heard it said.

The problem comes when future self decides to say something different to what past self remembers hearing.

For consistent time travel to work as simply as you say, you have to totally deny free will to anyone - and that doesn't make for a good RPG.

The way Continuum would handle that would be that by deciding to say something different to what your past self remembers hearing, you've fragged yourself - and it's a nasty frag too because there's no easy way to resolve the apparent paradox.

To get rid of the frag, you'd have to (or preferably arrange for someone else to) find yourself after the original conversation, and then alter your memories through hypnosis or tech means.

That would resolve the apparent paradox. You actually always said the thing that your elder self decides to say, and you only remember it differently because someone altered the memories of it that your younger self had.

Of course, since you don't remember having had your memories altered, that would have to be done surreptitiously.

Plus, if other people were there and heard what your elder had actually said they would have to have their memories altered accordingly so they also remember your elder saying the same thing that you remember them saying.

Plus, if you acted on what you remember your elder saying then whoever altered your memories would have to make sure that they did it in a way that didn't stop you acting in the way you did.

jseah
2011-11-24, 04:49 AM
I'm going to propose it to be the Path of Least Resistance.
The problem with this is that consistent time travel can only work in a universe that is deterministic.

Or at least deterministic in the quantum wave functions.

Determinism means that unless a future you has affected things, whatever would have happened without a future you will happen.

If you did not visit one week into the future to check yourself, it does not affect whether the bungee rope you chose would have a small nick and break when you jump with it.


Imagine a wave function that completely describes the state of everything in the entire universe. By smooth, I mean that there are no sudden changes in it (like there is in the step function) and there are no sudden changes in its rate of change (like there is in the absolute value function). An example of a temporal boundary condition is that at 4am CDT I was asleep; basically it is an association of a state with a time.
The time travel is basically a closed time-like curve. The universal wavefunction will interact with its future self when you time travel. It's not a cut/paste kind of time teleportation.
EDIT: sorry, yes, apparently I am confused. It IS time teleportation if you don't exist in all the intervening periods of time when travelling backwards.

-----------------------------------------------------------------------------

NichG & Infernal Bargain:
On the other hand, resolving which of the many consistent loops happen comes under quantum mechanics.

There was an interesting paper linked the wikipedia page I mentioned in the first post. That one calculated the trajectory of a billard ball through a closed time-like curve that would classically generate a paradox (knock itself out of the path leading to the wormhole)

They eventually concluded that no billard ball arrangement was not resolvable in a consistent manner by the future billard ball.
IE. there is no way to arrange things such that a paradox is inevitable. I am using this particular assumption.

Futhermore, they did a quantum sum over histories and found they could assign a probability of occurence to each individual path of the billard ball and all those that were inconsistent had a probability of 0.

And of course, if we are invoking the universal wavefunction, then in some sense, *every* single path with a non-zero probability happened. The billard ball is just entangled with its future self.


The "principle" says there is no paradoxes and there is only one timeline, yet it is so easy to create one in such a world as it could practically be said to be harder not to create paradoxes with time travel.
If you look at the billard ball example.

The billard ball is heading towards the time wormhole. If it isn't deflected, it will enter the wormhole in such a way as to knock itself off course. This is known.

If nothing comes out of the wormhole to deflect the ball, the ball will go in (and this is paradoxical already since nothing came out and something went in)
This paradox is not resolvable without invoking the future. Remember deterministic universe? (or the universal wave function interacts in a deterministic manner)
There won't be a chance deflection of the ball if there wasn't going to be one. Yes, you can use this to test if something will interrupt the time travel. Yes, this is information travelling backwards in time. Not at all surprising given we have time travel, no?

However, the authors of that paper think there is always a consistent solution and this solution involves the future billard ball.

In such a case of attempting to create a paradox, and in the case that chance events aren't going to stop you (if they were, it's not a paradox), then the only way to generate consistent loops is to have something arrive from the future that generates the consistent loop.

NichG
2011-11-24, 05:28 AM
The problem with this is that consistent time travel can only work in a universe that is deterministic.

Or at least deterministic in the quantum wave functions.

Determinism means that unless a future you has affected things, whatever would have happened without a future you will happen.


I think it'd be slightly easier in a non-deterministic universe. Basically, one could imagine inconsistencies that are small enough that they can be explained away by thermal noise. For instance, lets say you have a closed space-like curve for a single fleck of dust (I always get confused - spacelike curves are the ones that lie outside of the lightcone I thought, not timelike). This particular fleck of dust happens to only have 3 atoms in it, but because of an inconsistency it comes through with one of the atoms having an extra neutron. That path would have a zero weight in a deterministic universe (and the set of paths with non-zero weight that include time travel is probably of measure zero, but I can't prove that).

Now lets add some thermal or quantum indeterminancy with an energy scale E0 (this'd be proportional to hbar but I don't think its the Planck energy, which is huge; it probably has to do with the time that the inconsistency is required to persist before it can be corrected). Now the path has a weight exp(-E/E0) where E is the mass energy of the neutron (~940MeV). Thats still vanishingly small if we compare it to say the thermal energy at room temperature (0.025 eV or so), even for a neutron, but at least its non-zero.

So, not much easier, but a little bit :smallsmile:

Ravens_cry
2011-11-24, 11:27 AM
jseah
On billiard balls:
Which is where things get silly. What stops me, who? Not me, if I have free will. Not anyone else if they have free will.
If I simply drop dead, why wasn't my body discovered?
If you want less headaches, just go for alternative timelines. Of course, now you can only change one future and not your own and if the time line is quantum brittle you can never go back to your original time line without the ability to move sideways in time, but at least you don't have to worry about the universe playing temporal billiards.

jseah
2011-11-24, 12:07 PM
Part of the assumptions inherent in all this is that it is impossible to generate a situation that has NO consistent solutions.

Ravens_cry:
Your free will with and without time travel is still limited by circumstances. You can think of doing whatever you want, but whether you actually do it successfully (or at all) depends on whether you could.
You can't fly without wings. You can't cause paradoxes. They're not inherent limiters of free will.

The assumption in consistent-only time travel is that anything that would lead to inconsistency is impossible.

And chance events unconnected to the time travel aren't affected. A random meteor strike that would not happen if you never tried time travelling won't suddenly happen if you plan to create a paradox.
Unless the act of planning causes it or something arriving from the future causes it, random stuff just stays that way.

You might plan to do this basic paradox:
5 minutes from now, I will time travel from the future and tell myself a secret password. But when I actually time travel myself, I will mention a password that is different from the one I heard from my future self.

The you that arrives from the future would not simply fulfill the paradox as you plan since that would be... well, inconsistent.
The future you would be acting in such a way to cause you to act in a consistent fashion.


If you think about it, the amount of free will you have here is exactly the same as the situation of an advertisement convincing you to go buy a new pair of socks.

------------------------------------------------------------------------------------

That said, I am still trying to think of a pair of linked time loops that do something that can't be done with only 1 loop, however nested.

Maybe if I had two locked boxes that contain the other's key...

Ravens_cry
2011-11-24, 12:22 PM
Then it's even MORE screwed up. For the universe to maintain consistency, it breaks other laws, like in the explosive or even the meeting myself example.
You could prevent, say, a mass of uranium 235 from going supercritical by simple virtue of it happening in the past. After all, if a nuke went off in Vancouver on October 25th 2010, that would create paradox and since paradoxes allegedly can not be created, it could not happen.

jseah
2011-11-24, 12:33 PM
No. Violating physical laws is not consistent in the same way that something happening and not happening at the same time (paradox) is not consistent.
If you squirt neutrons at a critical mass of weapons-grade uranium, it will almost certainly blow up regardless of whether it creates a paradox in doing so.

You just won't be doing that. Part of the assumption, I have said many times, is that there is always a consistent solution.
Something comes back or forwards in time to cause a consistent solution, possibly enabling that time travel itself in the first place.

EDIT:
In the explosive example, you won't be detonating the explosive. You might not even be making the trip in the first place.

-possible solution-
You might get a visit from your future self who happens to need a piece of explosive for some urgent purpose (eg. defending himself against a terrorist) and he wants to use yours.

And that visit, in some manner causes those very same events that lead up to you in the future needing that piece of explosive.

Ravens_cry
2011-11-24, 12:43 PM
And that assumption is what I am getting at. Your 'solution' only works in a fictional universe as the characters in a story literally have no free will and so can have all sorts contrived circumstances happen to them and do as the Author says. Because why would past-me make use of that mass of explosives?
After all, if I am storing it with Past-me, a me with similar goals as myself, making the big explosion, Past-Me would not use that mass of explosives. Past-me would find some other source.

jseah
2011-11-24, 01:45 PM
Past you doesn't necessarily need that explosive. Would you rank a "paradox attempt" more important than "trying to stop a terrorist"?

Besides, your future self needs that explosive that you gathered. He didn't leave it with you, you made it for the purpose of doing this paradox attempt. He needs it for something else he explains, which is sufficiently urgent and convincing that you give it to him.

That's just one possible solution. If you can't be convinced, then it's not a consistent solution and you won't be seeing it happen. Something else happens.

Indeed, as with the billiard balls, it might be possible that an *infinite* number of solutions exist, although they might all be minor variations of each other.

Ravens_cry
2011-11-24, 01:56 PM
Nope, not if I am the type to set off explosion in the past in an attempt to create paradox. It doesn't have to be a damaging, it could be enough to get other peoples attention. Hell, signal flares and fireworks would work. It's just that permanently defacing something creates a very visual change.
To work around your 'solution', I tell myself if future me shows up looking for explosives, we gather explosives some other way. After all, if you are in the past, it's not like the future is going anywhere, there is no urgency, we can take our time.

jseah
2011-11-24, 02:45 PM
Or you give it to him and try again. Yes, I'll admit I can't think of air-tight solutions.

It's not hard to think up paradoxes. My earlier mention of the secret pass phrase that you get from your future and you resolve to tell yourself differently is a much simpler form of paradox and far harder to find a consistent solution.

You can even make sure to find a calm time in the past to try it.

The assumption there is that a solution is found and that is what happens. And yes, it is far easier to think up a hard to resolve paradox than resolve it.


This is the same way time loop logic works. You can make the paradox of getting the wrong answer, so as to ensure the answer you get is right.

Solving the problem (and thus the paradox) is incredibly hard. Its an NP-hard problem after all (eg. cracking RSA encryption).
In the same way as being incredibly hard to resolve the RSA problem, it is hard to resolve such paradoxes for a writer or GM. We don't have time travel to enforce consistency and hand us the answer.

The assumption here is that such solutions *always* exist and one of them will happen.
Something appears from the future and resolves the situation in a consistent, non-paradoxical manner.

Ravens_cry
2011-11-24, 02:53 PM
And that is why I consider the Principle, for lack of a better word, silly.

Shadowknight12
2011-11-24, 03:41 PM
Why would you go with this? Consistent time travel is logically impossible anyway. The only type of time travel that makes logical sense (but presents a bleak, bleak outlook on the setting) is deterministic time travel. I.e., you travel back in time and do whatever, and that was exactly what you were meant to do. I'd personally hate to have a setting with such a dystopian outlook. Everything is objectively predetermined? Shudder.

NichG
2011-11-24, 04:13 PM
There is always the null timetravel solution to any issue with self-consistent timetravel and paradox creation. Basically anyone who would create a paradox never receives the opportunity to time travel. If one insists absolutely on free will, this would logically mean that no person ever time travels or has sufficient control over time travel phenomena to attempt to modify the past. Logically I suppose this would mean that deterministic time travel mechanisms (i.e. you can, without first having time travel, create a situation in which backwards time travel will always occur) must be hard or impossible to access.

In any event all the theorized deterministic means of time travel I'm aware of (i.e. wormholes outside of eachothers' light cones) have energy divergences in their solutions.

Non-deterministic time travel has the 'advantage' if you will that in cases where there's no resolution it can always default to 'it simply doesn't happen this time'. If we imagine such a thing as being real, it would almost certainly be limited to isolated sets of one or two particles over femtosecond timescales where inconsistencies can be absorbed by virtual particles. The probability of the time travel event occurring would be proportional to the density of time travel solutions compared to non-time travel solutions, so you'd only get it when the NP problem isn't that hard.

Edit: Just read the billiards paper. I guess this shows that the set of time travel trajectories aren't a set of measure zero in the deterministic case, but actually dominate!

jseah
2011-11-24, 04:40 PM
The only type of time travel that makes logical sense (but presents a bleak, bleak outlook on the setting) is deterministic time travel.
Um, you do realize that one of the other assumptions of this consistent time travel is that the universe is deterministic?

Or at least the quantum wavefunction is deterministic.

So yes, this is a deterministic time travel that is restricted to consistent time loops.

Why would you say this is a bleak outlook? Determinism has nothing to do with time travel, save that consistent time travel requires determinism.

Raven's Cry:
But as implied in the billiard balls example, no such arrangement of balls can force a paradox.

Why not in the macroscale as well? There isn't anything fundamentally different when it comes to people, just that the problem has become many orders of magnitude harder.

The billiard balls problem suggests that solutions to such apparent paradox setups do exist. And in the same way that people are more complex than billiard balls, the solutions to those problems are likewise more complex.

And yet, people don't follow any physical laws that billiard balls don't (and vice versa), which indicates that the same principles apply.
Namely, consistent solutions always exist.


After all, determining such a solution is one example of an NP-hard problem and time loop logic can solve it.
If you think about it, setting up a paradox situation is akin to posing a NP-hard problem to time loop logic. The problem here is just "given the particular state of the universe, what future arrival could result in a consistent time loop?"

NichG:
The kind of virtual particles that could be generated without being "inconsistent" are those below the Heisenberg limit.

In any case, I'm not talking about a hypothetical time machine. The kind of problem I am asking here assumes that "you" can time travel in the same way superman can fly.

All:
You do realize that I'm not asking for answers to questions that arise from this. Nor am I even asking for the questions themselves.

I am just looking for more complex applications of the time travel power. More complex than those I just put up that is.

While I do acknowledge that many questions and problems are put up by my definitions of how this time travel superpower works, I am not looking for them.
I am aware of the implications on determinism and have thought about the sorts of experiments you can run with this. (one of them is to determine if there is a global hidden variable behind quantum mechanics. The Bell Test Experiments rule out local hidden variables, but not global ones)


If superpowers like telekinesis and invulnerability can receive the rule of cool handwavium, I appeal that the same principle applies to my question.

If you think that our universe is not at least deterministic for quantum wavefunctions, I would disagree. But I am not making this thread to argue that point. For the purposes of considering this time travel power and its applications, the universe where this is possible IS deterministic and consistent solutions *always* exist.

NichG
2011-11-24, 04:54 PM
Okay, fair enough.

So here's a question for you. In the billiards example, does the presence of an extra stationary ball somewhere else in the space change the set of allowed solutions (i.e. damping out a set of paths that would require free travel through that space to be self-consistent)? If so, you could use self-consistent time travel to obtain information about regions in space that you 'could-will-have' passed through but won't.

E.g. lets say you're piloting a craft through an asteroid field, and you know the location of a wormhole mouth that could be very far away but has a relatively clear line of sight. Now, you want to exploit the fact that a year ago, that line of sight was blocked by the asteroids (or a day ago or whatever). So what you do is you fire a probe towards the wormhole mouth. It must self-collide since this is a dangerous trajectory, so what happens is that there is another version of the probe (or several) on ballistic trajectories that safely pass through the asteroids so they can get to the probe to self-collide. By watching the subsequent collision, you can determine a safe straight-line path through the asteroids.

Shadowknight12
2011-11-24, 05:54 PM
[SPOILER]Um, you do realize that one of the other assumptions of this consistent time travel is that the universe is deterministic?

Or at least the quantum wavefunction is deterministic.

Your OP didn't really spell that out. Or, failing that, I think we're not talking about the same things.


So yes, this is a deterministic time travel that is restricted to consistent time loops.

Then you can't actually formulate 'uses' for it, since everything that time travel facilitates already needed time travel to happen, so you're not getting any "use" out of it. In other words, time travel is not as advantageous as it is a necessity, in that you don't really have a choice about it. If it's determined that your time travel skit will benefit you, then that's what's going to happen. If it's determined that it won't, tough luck.


Why would you say this is a bleak outlook? Determinism has nothing to do with time travel, save that consistent time travel requires determinism.

It's a bleak outlook on the setting, because it implies that there's no free will. Everything that happens is already pre-determined, your choices don't matter (because anything that you think has been the result of your own choosing was in fact what you were meant to do all along) and time travelling becomes a necessity, a chore, rather than an adventure.

If I actually have to explain to you why an absence of free will is a bleak and depressing outlook on a setting, I think we have such fundamentally different life philosophies that we could never understand each other in the first place.

jseah
2011-11-24, 06:01 PM
In the billiards example, does the presence of an extra stationary ball somewhere else in the space change the set of allowed solutions (i.e. damping out a set of paths that would require free travel through that space to be self-consistent)? If so, you could use self-consistent time travel to obtain information about regions in space that you 'could-will-have' passed through but won't.
Hmm...

I do believe both of those statements is correct!

Assuming "It must self-collide since this is a dangerous trajectory" means that if the probe does not self-collide, results in a paradox.

In fact, you can do this for probing the future too.

"In two days time, I will send back a piece of paper saying 'all clear'. "

If you do get that piece of paper, not only does it say that the paper got sent, it also implies that you were able to send that peice of paper (ie. does not cause inconsistencies and your time travel power is still working)

In this way, you could set up paradox-causing situations that can resolve one way or another depending on the answer to a question. One that you don't know the answer to.

Which, although extremely similar to the time loop logic application, can probe simple questions across any point in time and space reachable in the time period of the proposed loop.
And you don't have to prove the answer correct, unlike in time loop logic.

A billiard ball version of your problem goes like this:
"There's a billiard table and a ball on it. The path the ball is going through the time wormhole if it does not collide with anything will result in the ball colliding with itself. This is inconsistent.

There are a number of potholes in the table. If the ball rolls over a pothole, it stops. Other than a ball stopping over a pothole, we have no way to tell where those potholes are.
The inconsistent path is known not to have potholes. "

Assuming a path that does not go over potholes exists that is consistent. (otherwise seriously weird stuff can happen, like trick shots around rims of the pothole. I don't think its possible to generate a billiard table that has NO inconsistent solutions. )

You can find that the consistent path that eventually does happen somehow "knows" where the potholes are.
And if you watch the ball, you might find out where the potholes are without having the ball fall in them.

jseah
2011-11-24, 06:40 PM
Your OP didn't really spell that out. Or, failing that, I think we're not talking about the same things.
I apologize. Yes, that was not included in the OP although I did mention it a few posts back. IIRC, it was to NichG.

EDIT: I have edited the OP to include these two assumptions.


Then you can't actually formulate 'uses' for it, since everything that time travel facilitates already needed time travel to happen, so you're not getting any "use" out of it.
You still need to formulate how to use it. Let's say there is an extremely complex application of this time travel thing.

Multiple interlocking loops, ontological "paradoxes" out the ears; time loops nested twenty deep, a few hundred copies of you running around.
Let's say you need to do that in order to do some especially complicated thing. I dunno, maybe steal a nuke? When you're a 14 year old kid who knows nothing about how anything actually works.

So. If you didn't know how to execute this incredibly complicated loop, you'd have to learn it from your future self or fail.

Without actually formulating how the thing works, either the normal way or by ontological paradox (future teaching), you'll just fail.


It's a bleak outlook on the setting, because it implies that there's no free will.
I don't see how determinism implies
Ok, I DO see how deterministic universes can be argued to curb free will.

To me, deterministic universes do not rule out free will, but perhaps how we define free will is different.

However, this has no bearing on the time travel itself.

Shadowknight12
2011-11-24, 07:26 PM
I apologize. Yes, that was not included in the OP although I did mention it a few posts back. IIRC, it was to NichG.

Might want to add it to the OP to avoid further confusion, then! :smallsmile:


You still need to formulate how to use it. Let's say there is an extremely complex application of this time travel thing.

Multiple interlocking loops, ontological "paradoxes" out the ears; time loops nested twenty deep, a few hundred copies of you running around.
Let's say you need to do that in order to do some especially complicated thing. I dunno, maybe steal a nuke? When you're a 14 year old kid who knows nothing about how anything actually works.

Doesn't matter. That's the thing about determinism. You cannot fail. You don't need to understand anything, as a character, because you will do whatever you need to do regardless of whether you have any intention or ability to do so.

As a writer, you just need to figure out what's going to happen beforehand (i.e., arrange all the loops and paradoxes) and write it.


So. If you didn't know how to execute this incredibly complicated loop, you'd have to learn it from your future self or fail.

Without actually formulating how the thing works, either the normal way or by ontological paradox (future teaching), you'll just fail.

Nope. You don't fail. You can't fail in a deterministic universe. That's why I think we're not talking about the same thing. You seem to have a different definition of 'deterministic' than I do. The way I see it, this discussion can't be discussing time travel from the characters' point of view, since none of their decisions matter. They can't 'game' time travel. They will do exactly what they were meant to do, nothing more and nothing less. This conversation is useful, though, if you need plot ideas as a writer.



I don't see how determinism implies
Ok, I DO see how deterministic universes can be argued to curb free will.

To me, deterministic universes do not rule out free will, but perhaps how we define free will is different.

However, this has no bearing on the time travel itself.

It doesn't "curb" it, it eliminates it entirely. If you travel back in time, you will have always been meant to do that, and whatever you do there will have always been necessary. Whether it benefits you or not is a secondary concern at best. You are a cog in the machine of the universe, performing your function without even realising it. And because the universe is objectively deterministic, you can't even rebel. You can't "malfunction" or stop turning like a good little cog. You stopping means that you were meant to stop all along, so your little tantrum only furthers the will of the universe even further.

To me, when the outcome of your decisions has already been pre-determined, and you cannot choose any other path but the one that was laid before you by someone else, you have no free will. You are quite right in that we might be defining such a loaded concept differently.

NichG
2011-11-24, 08:55 PM
The part I was confused about (but the billiards case clarified for me) had to do with gaming time travel, so it might be useful to spell it out.

Essentially you can game time travel if you have a way to (deterministically) guarantee that time travel occurs. If you can do this, then in some sense the universe 'must' select a way for time travel to occur that is consistent. The trick with the billiards is that you can create a situation where 'no time travel occurs' is inconsistent, forcing some form of time travel to happen.

This is basically covered by some (preferably macroscopic) opening into another time. A 'device' that sends you back in time is not going to be as good for gaming time travel because it can easily fail or do something different due to microscopic influence. Time travel occurred, but its not controllable or usable since its just butterfly-effecting its way to consistency. This could be a problem with 'personal timetravel' powers - whatever mechanism allows for time travel could trigger to force consistency in a microscopic and uncontrolled fashion.

On the other hand, if you have a big opening such that anything that passes through it will end up in the past and you lob a ball at it, there's not much in the way of microscopic time travel that will rectify that trajectory. Whatever the response is will be as big as the inevitability you've created. Even if something occurred to stop you from lobbing the ball, it itself would be an instance of usable timetravel and would have to have its own source of consistency. So in this case, you can extract benefit from it despite it being deterministic.

The important element is that you must be able to create a situation that cannot be resolved either by 'no time travel occurs' or by 'microscopic time travel butterfly-effects things and genies your attempt at time travel'. So a stable wormhole that is always on is going to be a lot more useful than any situation where time travel could end up not happening.

Velaryon
2011-11-25, 02:01 AM
It's easy to counter: the best part of using secret passwords while contacting yourself is, that you never have to share it with anyone. If you contact your future self, you don't even need to have a set password - you can make one up right there and the future you should know it for obvious reasons.

Yes, the system isn't ideal, but it gives you a serious advantage.

I think this (http://www.youtube.com/watch?v=zYWT4uYOPvs) is relevant here.


Sounds like Bill and Ted: The Game. WANT!

Exactly!

Infernalbargain
2011-11-25, 02:23 AM
And chance events unconnected to the time travel aren't affected.

Actually I disagree. Consider the following set up: you have a red button and a green button connected to a red light and a green light, a fair coin, and a time machine. When you push one of the buttons, there is a probability, K (K is a number between 1/2 and 1), that the corresponding light will turn on in the future, otherwise the other light turns on. So our probability matrix for the lights is as follows:
[ K 1-K
1-K K ]
Before you press the button, you go into the future to see which light turned on. Then you flip your coin twice. If both times it comes up heads, you go back and push the button that is the same color as the light that turned on. Otherwise, you go back and push the button that does not correspond to the light. To make this brief, I will use the following notion THRG means that the first coin came up tails (T), the second came up heads (H), the light was red (R), and the green button was pushed (G).
HHRR : K*1/4
HHRG : 0
HHGR : 0
HHGG : K*1/4
HTRR : 0
HTRG : (1-K)*1/4
HTGR : (1-K)*1/4
HTGG : 0
THRR : 0
THRG : (1-K)*1/4
THGR : (1-K)*1/4
THGG : 0
TTRR : 0
TTRG : (1-K)*1/4
TTGR : (1-K)*1/4
TTGG : 0

These are all the possible outcomes. Now our elementary math classes tell us that the total probability is 1. Summing all this up you get 3/2 - K, which is not necessarily 1. A few other fun things crop up, your coin is no longer fair in the presence of a time machine, since the probability of HH coming is K*1/2, not 1/4. The astute among you might have noticed that in the limit of K->1/2, the effects of the time machine go away; we get a probability cross-section of 1 again and our coin is fair.

So now the question is what the hell happens when K != 1/2? In order to preserve our classical notion of total probability = 1 means that something must happen other the seemingly comprehensive above scenarios with a probability != 0. The only solution is that something completely unrelated happens. Since there are no constraints on what the result of the "missing" probability, we have no idea what is going to happen when you push the button.

jseah
2011-11-25, 07:31 AM
Might want to add it to the OP to avoid further confusion, then! :smallsmile:
Already done!


Doesn't matter. That's the thing about determinism. You cannot fail. You don't need to understand anything, as a character, because you will do whatever you need to do regardless of whether you have any intention or ability to do so.
Remember than in determinism, all things have a cause and effect such that the causes fully define their effect. Those effects then go on to become causes for the next effect and so on.

Dropping a ball causes it to hit the ground. If you don't drop the ball, it will just stay in your hand. No physical laws are violated.

Likewise, if the only time traveller existing is you, and if you don't know how to execute the loop and you aren't learning it from your future self, then you will not be executing that loop.
Because to do so would be an effect without a cause.


If humans only do things when they decide to do so, then deciding to do something is a necessary cause for some action you take. Sure, you were always going to do it because your decision is an effect that has its own cause-effect chain that is deterministic. But you still have to / will be making that decision anyway.

Likewise, you can fail in a deterministic universe if you were going to fail.


To me, when the outcome of your decisions has already been pre-determined, and you cannot choose any other path but the one that was laid before you by someone else, you have no free will. You are quite right in that we might be defining such a loaded concept differently.
The outcome of your decisions and the decisions themselves are both pre-determined and with time travel, you can find out what they were.

You will be choosing X because of Y reasons.
You will be doing Z because of X and you will succeed/fail due to yet more reasons.

It's not that you *cannot* choose other paths. You *won't* choose other paths. There are fully real and, with effort, knowable reasons why you made any particular decision.

The important element is that you must be able to create a situation that cannot be resolved either by 'no time travel occurs' or by 'microscopic time travel butterfly-effects things and genies your attempt at time travel'. So a stable wormhole that is always on is going to be a lot more useful than any situation where time travel could end up not happening.
Yes, that would be a good guideline on how to use time travel effectively.
Although I don't think you require a stable wormhole.

A person with a time travel superpower that can only transport him/herself plus a bit, no more or less, would force all time travel into the macroscale.


Actually I disagree.
<...>
Before you press the button, you go into the future to see which light turned on. Then you flip your coin twice. If both times it comes up heads, you go back and push the button that is the same color as the light that turned on. Otherwise, you go back and push the button that does not correspond to the light.
You want a machine to flip your coin and take very much care to not affect your machine. But yes, this is essentially a paradox situation.

The coin and machine and you are macro-scale objects and thus their quantum fluctuations can be ignored.
In a deterministic universe, the future would already have been affected by the coin toss. Assuming your time travel event cannot affect the toss itself, then that toss is effectively pre-determined.

Whether you get head-head or another combination is not affected by your time travel.

If you get head-head, then there is no paradox, and you see a green or red light and push the corresponding button.
If you aren't going to get head-head, then it becomes a standard paradox situation and some other stuff will happen to make it consistent.


The reason why I would suggest you aren't the one to flip the coin is because what you see in the future could potentially affect how you flip the coin when you get back.
It would not be too hard to imagine some minor variation that influenced you in a microscopic manner to end up flipping head-head.
*** This is what NichG meant by "microscopic time travel butterfly-effects things and genies your attempt at time travel" ***

jseah
2011-11-25, 12:17 PM
I have been thinking about differentiating the types time loops (including non-loops and trivials) that you can do. So here's my first shot at it. Perhaps this might point out a guideline to a more complicated application of time travel.

Non-travel (trivial)
Non-time travel. Default.

Skip (trivial)
Time travelling to the future. Does not include return trip.
You do not exist in the intervening time period. No paradoxes possible from this trip alone.

Non-paradoxical backwards time travel (non-loop)
Time travelling backwards to a point that the time travel event is outside the light cone of your destination. Might require FTL.
Low to no chance of paradox as you can't affect anything that caused your time travel.

This can morph into a paradoxical time travel situation if there is a Skip in the subjective past of this time travel event.

Trivial "paradox"
You time travel backwards to a point that can affect the time travel event. Technically paradox-causing time travel.
However, you don't act in any way to actively cause a paradox. Consistent solution is still enforced, but effects of enforcement are all on the microscopic unseen level.

Pre-determined travel paradox
If nothing arrives from the future, you will be travelling backwards to a point such that you could affect the time travel event itself. Therefore, all non-time travel solutions are inconsistent.
The solutions to time loop logic and attempted paradoxes fall under this category. Information or things from the future ensure a consistent solution is reached.

Basic ontological loop
Non-time travel solutions are consistent. A future you appears and performs a set of actions that enables you to time travel in the first place.
Teaching yourself time travel, future self saving yourself from rogue gunmen and teaching yourself how to use a time travel loop all fall under this category.

Interlocking ontological loops
Non-time travel solutions are consistent. More than one future yous appear and perform actions that allow each other's and your time travel to happen. Solutions involving time travel as intended must have all the loops to be consistent.
eg.
Two buttons unlock doors to the room that contains the other button. Two future yous appear in the rooms and press the buttons. Present you goes into one room, that room's future you (first iteration) goes to the other room, the other room's future you (second iteration) comes out and is the one that remains when present you and first future you travel backwards.

Material non-ontological loop
A piece of material you bring with you in your time travel interacts with its past self to form some useful object. Possibly travelling many loops through the same time travel event before exiting.
Eg. One-atom key

Material ontological loop
A thing from the future appears and ends up being the same thing that is sent back. Such a thing has no beginning and no end.
Theoretically possible, thermodynamically extremely unlikely. Probability of such a thing goes to zero to the permutations of identical arrangements of that object's particles.
The object would have to be restored to the exact state at which it arrived before it time travels backwards.

Theoretically, it would be possible to engineer a pre-determined travel paradox whose only solutions involve basic or even interlocking ontological loops. Blocking those would be presumably impossible as they can do practically anything.

Consistent resolutions to some more air-tight paradoxes might require more than one time travel event.
This could be a good way to force an otherwise incredibly unlikely material ontological loop.

Shadowknight12
2011-11-25, 02:38 PM
Remember than in determinism, all things have a cause and effect such that the causes fully define their effect. Those effects then go on to become causes for the next effect and so on.

That's not determinism. That's causality. I am growing ever more convinced that we're discussing completely different things.


Dropping a ball causes it to hit the ground. If you don't drop the ball, it will just stay in your hand. No physical laws are violated.

But in determinism, it is already predetermined whether you will drop the ball or not, and if so, when and how. Determinism isn't about physical laws, it's about the illusion of choice. You don't have a choice when it comes to determinism. What you did is what you were meant to do all along.


Likewise, if the only time traveller existing is you, and if you don't know how to execute the loop and you aren't learning it from your future self, then you will not be executing that loop.
Because to do so would be an effect without a cause.

That is factually wrong if we're discussing a deterministic time-travel setup. In determinism, if you don't know how to execute the loop, you will end up executing it entirely by accident, or even against your will. That's the essence of determinism, the fact that events are predetermined and you cannot change them in the slightest. Even your time-travel skit was predetermined, and everything you do there was meant to happen all along.

As for the "because to do so would be an effect without a cause," again, you're talking about causality. What you say makes perfect sense when it comes to causality. Determinism is far, far more than mere causality.


If humans only do things when they decide to do so, then deciding to do something is a necessary cause for some action you take. Sure, you were always going to do it because your decision is an effect that has its own cause-effect chain that is deterministic. But you still have to / will be making that decision anyway.

No, because it doesn't matter if you make the decision or not. Determinism states that whatever events must happen, will happen, regardless of the desires or intentions of anyone involved. Whatever decision you're making is the right one because it's the one that you were meant to make all along.


Likewise, you can fail in a deterministic universe if you were going to fail.

Only if you define 'fail' according to the character's perceptions. If you determine 'fail' from an objective perception, then no, you cannot fail, because you will always succeed at doing what you were meant to do all along.


The outcome of your decisions and the decisions themselves are both pre-determined and with time travel, you can find out what they were.

You will be choosing X because of Y reasons.
You will be doing Z because of X and you will succeed/fail due to yet more reasons.

It's not that you *cannot* choose other paths. You *won't* choose other paths. There are fully real and, with effort, knowable reasons why you made any particular decision.

But it doesn't matter. It doesn't matter what your reasons for doing X was, because you never had a choice to doing anything but X. It's like being asked at a restaurant if you'd like chicken or chicken. You can choose chicken because you like it or because that's the only choice you had, but it won't matter, because at the end of the day, you'll still be having chicken.

And the last paragraph is just plain wrong. It's true, you won't choose other paths, but determinism means that you cannot choose another path, or else the universe isn't deterministic.

jseah
2011-11-25, 04:03 PM
Rather than spoilering each section, you'd save more space by spoilering the whole post. =P

That's not determinism. That's causality. I am growing ever more convinced that we're discussing completely different things.
And determinism is just an assertion that the causes fully define the effects. Given an accurate state of the universe and an accurate set of physical laws, determinism is the assertion that those are enough to fully define all future states of the universe.
(the ability to also define all past states depends on the laws being time symmetric, which they almost certainly are)
Those laws and the current state is a sufficient and complete description of causes that lead to the effect, which is the 'next' state of the universe at some future point in time.
Current state of universe + physical laws = total definition of the universe (all points in time included)

I'm just rephrasing it here. The explanation about causes defining effects is to me, identical to saying "the universe is deterministic".

But in determinism, it is already predetermined whether you will drop the ball or not, and if so, when and how. Determinism isn't about physical laws, it's about the illusion of choice. You don't have a choice when it comes to determinism. What you did is what you were meant to do all along.
Uh. Ok, now I'm not sure we are even talking about the same thing anymore.
That choice is also part of the universe and is also determined by those same physical laws. The fact that you chose to drop the ball rather than not is theoretically traceable to well-defined causes, and a different set of causes might have resulted in you deciding to keep holding it instead.

There's no "what you were meant to be doing". Meaning doesn't exist. Nothing is "meant to be".
There's only "what the future is".

That is factually wrong if we're discussing a deterministic time-travel setup. In determinism, if you don't know how to execute the loop, you will end up executing it entirely by accident, or even against your will. That's the essence of determinism, the fact that events are predetermined and you cannot change them in the slightest. Even your time-travel skit was predetermined, and everything you do there was meant to happen all along.
No.
What you know and don't know is also part of the universe and also governed by the physical laws. If the loop *requires* that you know how to do it to execute the loop, then the *only* way the loop gets executed is if you know how to do it.
That is a logical necessity.

Of course time travel skits are pre-determined. You can affect the past (because that's what time travel is) but your effect on the past is in the past and doesn't change anything. Because whatever you did has already happened in objective time.
Change is not something that happens in a deterministic universe.

As for the "because to do so would be an effect without a cause," again, you're talking about causality. What you say makes perfect sense when it comes to causality. Determinism is far, far more than mere causality.

No, because it doesn't matter if you make the decision or not. Determinism states that whatever events must happen, will happen, regardless of the desires or intentions of anyone involved. Whatever decision you're making is the right one because it's the one that you were meant to make all along.
No. The desires and intentions of people are part of the universe. They are part of the causes and contribute to the effects.
If people did not have the exact desires and intentions as they did in this state of the universe, then in this hypothetical "identical in every way except for this guy's desires" universe would play out differently.

The differences would then butterfly effect their way into generating a different future state.

Only if you define 'fail' according to the character's perceptions. If you determine 'fail' from an objective perception, then no, you cannot fail, because you will always succeed at doing what you were meant to do all along.
There is no "what you were meant to do". Failure and success can only be defined from a human perspective, the universe just is and whatever happens, be it failure or success or something unclassifiable, merely happens.

Current state + future laws = you fail
Different state + future laws = you succeed
That's it.

But it doesn't matter. It doesn't matter what your reasons for doing X was, because you never had a choice to doing anything but X. It's like being asked at a restaurant if you'd like chicken or chicken. You can choose chicken because you like it or because that's the only choice you had, but it won't matter, because at the end of the day, you'll still be having chicken.

And the last paragraph is just plain wrong. It's true, you won't choose other paths, but determinism means that you cannot choose another path, or else the universe isn't deterministic.
I think the earlier bits are enough. The reasons matter.
Perhaps we define "choice" differently.
When I say "you can choose", I mean this:
Among all hypothetical possible states of the universe in which a human would recognize that you are deciding a similar problem, some of those universes would contain a future in which you do something you did not in the real universe.
"you" is defined by: something that a human would recognize, using all human-scale metrics including anatomical, behavioural and observational, to be identical to the real life "you"
"what you do" is similar to the definition of you, but applied to an event. In this case, a decision. If a human recognizes that the decision you made is identical, then its not a "different" choice.

When I say you have a choice, I include all hypothetical universes that might have slightly different states. Not just the currently extant one.
Yes, those hypotheticals don't happen because they are not the current universe. But its a useful model and simplification as we *cannot* measure the current state of the universe with infinite accuracy.
So we have to consider all those similar states that fall within the errors we have (and human-scale metrics have huge errors) in order predict what could happen. Sometimes, this leads to us considering two or more noticeably different future states. And when this is due to us not being able to predict how a human would decide a problem, we call this a "choice".
Even though "choice" does not exist, it is a meaningful definition as it invokes hypotheticals that represent our lack of confidence in our predictive power.
Sure, it's only an illusion of choice as our choices are pre-determined. But that illusion exists due to our lack of ability and that lack is not going away just because we think the universe is deterministic.

Likewise, the illusion remains and is still useful both as a descriptive term and a functional one. Since we can't even *describe* the universe to infinite accuracy, or even all of it at nearly no accuracy, then referring to any description includes a huge number of hypotheticals. Describing branching into major differences due to human decisions is called a choice.

You seem to think that the universe doesn't include human choice or knowledge. Am I reading this right?

NichG
2011-11-25, 04:50 PM
You're presenting determinism as if its some sort of external puppeteer entity with its own desires, that it molds all actions into. That's a lot of anthropomorphizing.

A better way to think about it IMO is to look at history. Once an event is in the past, you cannot choose for it to have been differently - thats how it was and will have always been. It's not like the actors in the past are constantly trying to change what they did in ways invisible, and some external force is going and 'correcting things' so that they always do what they must do. No, its just that those events have occurred and are fixed.

Now take that and imagine sitting at the end of time and looking back at the universe. From that point of view, everything is in the past. That's not even determinism yet by the way, thats just static history.

Determinism is the connection where one says 'if we could have complete knowledge of the present then we would know what that end of time point of view snapshot looked like'. However, with incomplete knowledge (and in principle any missing knowledge inside the light cone of an event) then an event cannot be absolutely predicted. This is where ideas of free will come in - its hard to have complete knowledge of a person or their situation (or their neurochemistry for that matter), and so humans behave in unpredictable ways.

Time travel loops make that future snapshot visible earlier, thus bypassing the need to predict. However, they do not give information about the snapshot after the end of the loop or before the beginning of the loop.

Shadowknight12
2011-11-25, 10:29 PM
Rather than spoilering each section, you'd save more space by spoilering the whole post. =P

Kk.


And determinism is just an assertion that the causes fully define the effects.

No, that's not what determinism is. That's causality:


cau·sal·i·ty   [kaw-zal-i-tee]
noun, plural -ties.
1.
the relation of cause and effect: The result is the same, however differently the causality is interpreted.
2.
causal quality or agency.

Determinism:


de·ter·min·ism   [dih-tur-muh-niz-uhm]
noun
[...]
1. Compare free will Also called: necessitarianism the philosophical doctrine that all events including human actions and choices are fully determined by preceding events and states of affairs, and so that freedom of choice is illusory

Emphasis mine. Determinism is more than simply saying that all effects have causes. It's saying that all events, including human choices and decisions, are fully predetermined. If they are fully predetermined, nothing you can do can change them. You cannot change the present by going back in time and you can't change the future by going there, then going back to do something different. That's the essence of determinism and the only way it can work logically and without paradox.


Given an accurate state of the universe and an accurate set of physical laws, determinism is the assertion that those are enough to fully define all future states of the universe.
(the ability to also define all past states depends on the laws being time symmetric, which they almost certainly are)
Those laws and the current state is a sufficient and complete description of causes that lead to the effect, which is the 'next' state of the universe at some future point in time.
Current state of universe + physical laws = total definition of the universe (all points in time included)

That's the epistemological use of determinism. I'm referring to the philosophical one (see above), since we're discussing time travel, not a branch of science.


I'm just rephrasing it here. The explanation about causes defining effects is to me, identical to saying "the universe is deterministic".

And that's where you're wrong. That. Is. Causality.


Uh. Ok, now I'm not sure we are even talking about the same thing anymore.
That choice is also part of the universe and is also determined by those same physical laws. The fact that you chose to drop the ball rather than not is theoretically traceable to well-defined causes, and a different set of causes might have resulted in you deciding to keep holding it instead.

No. Saying that effects can be traced back to well-defined causes is causality. Determinism goes a step further and tells you that the chain of causes and effects cannot vary and cannot be changed. That's the entire point of determinism. Without the part about how they cannot vary, it wouldn't be determinism in the first place.


There's no "what you were meant to be doing". Meaning doesn't exist. Nothing is "meant to be".
There's only "what the future is".

No.
In determinism, there is most certainly a set chain of causes and effects, which is the one that is going to happen no matter what. I use 'meant' for the lack of a better word to express how unavoidable the situation is.


No.
What you know and don't know is also part of the universe and also governed by the physical laws. If the loop *requires* that you know how to do it to execute the loop, then the *only* way the loop gets executed is if you know how to do it.
That is a logical necessity.

This is patently false in several ways.

Firstly, knowledge is not governed by physical laws. It has no mass, it cannot be measured, it cannot be studied with the same tools we use to study the rest of reality, it cannot be located in time and space, and our ways to physically influence all require messing up a brain. Which we don't even know if it actually does something to the knowledge itself or the person's ability to recall and perceive it. So for all we know, knowledge itself is beyond our ability to influence physically.

Secondly, your logic necessity is false because no chain of cause and effects or loop requires knowledge. Reality cannot require knowledge. It requires actions, and they can be performed without knowledge. Go ahead, think of any action that can only be performed by a person with knowledge of the action, and not by someone by complete accident. Do keep in mind that things with an extremely low possibility of happening by accident (like a person stumbling over a tray with surgical equipment, sending a scalpel flying through the air and it landing on a patient's open thoracic cavity in such way that it neatly excises a tumour) are still things that could well happen without the requirement of human knowledge, and therefore continue to prove your 'logical necessity' false.


Of course time travel skits are pre-determined. You can affect the past (because that's what time travel is) but your effect on the past is in the past and doesn't change anything. Because whatever you did has already happened in objective time.
Change is not something that happens in a deterministic universe.

No no no no no. You're right, but at the same time, you're missing a key element of deterministic time travel. You cannot change the past. Whatever you do in the past is something that was necessary to happen, and therefore you are not changing anything, but doing exactly what needed to to be done for the future to unfold.

Example: Your parents are murdered by a stranger in front of your eyes when you're a kid. You devote your life to undoing that mistake. You go back in time to save them. Whatever actions you perform while you are back in time only further the chain of events that causes your parents' deaths. Maybe you forgot that a stranger tried to warn them not to go into the alley, and that stranger was future!you. Maybe you decide to viciously stab whoever follows your parents, but you get the wrong couple and end up killing your parents yourself. Maybe you try to intervene but you get hit by a bus and die. And your younger self didn't notice the death of the stranger outside the alley because he was too busy being traumatised for life by the death of his parents.


No. The desires and intentions of people are part of the universe. They are part of the causes and contribute to the effects.

That is very much debatable and not the clear-cut assumption you're making it out to be. "Desires" and "intentions" cannot be measured like we measure concrete, physical things. So while I can't prove that you're wrong about that, you can't prove that you're right either, because we cannot produce the necessary evidence to further our assertions. And as we both know, absence of evidence is not evidence of absence.


If people did not have the exact desires and intentions as they did in this state of the universe, then in this hypothetical "identical in every way except for this guy's desires" universe would play out differently.

The differences would then butterfly effect their way into generating a different future state.

Again, that is debatable. Determinism doesn't state that your desires and intentions are predetermined. It says that your actions and decisions (i.e., things that have observable effects) are. It's unknown if a deterministic universe actually cares what your intentions are or not. For all we know, you may well have every intention to do things differently but end up doing exactly the same, with polar opposite intentions.


There is no "what you were meant to do". Failure and success can only be defined from a human perspective, the universe just is and whatever happens, be it failure or success or something unclassifiable, merely happens.

Current state + future laws = you fail
Different state + future laws = you succeed
That's it.

Yeah, but we're not talking about our own universe. We're talking about a fictional universe. Therefore we have two viewpoints for this matter: In-character viewpoint and out-of-character viewpoint. In-character you're right, failure and success are decided by their perspective. But from an out-of-character perspective (us as authors, rather than us as characters), we decide what success and failure is. And to me, 'failure' would be to go back in time and fail to do the actions that took place originally. In a deterministic universe, that's not possible, so you end up with a zero chance of failure (from an out-of-character perspective).


I think the earlier bits are enough. The reasons matter.
Perhaps we define "choice" differently.

Perhaps we do.


When I say "you can choose", I mean this:
Among all hypothetical possible states of the universe in which a human would recognize that you are deciding a similar problem, some of those universes would contain a future in which you do something you did not in the real universe.
"you" is defined by: something that a human would recognize, using all human-scale metrics including anatomical, behavioural and observational, to be identical to the real life "you"
"what you do" is similar to the definition of you, but applied to an event. In this case, a decision. If a human recognizes that the decision you made is identical, then its not a "different" choice.

When I say you have a choice, I include all hypothetical universes that might have slightly different states. Not just the currently extant one.
Yes, those hypotheticals don't happen because they are not the current universe. But its a useful model and simplification as we *cannot* measure the current state of the universe with infinite accuracy.
So we have to consider all those similar states that fall within the errors we have (and human-scale metrics have huge errors) in order predict what could happen. Sometimes, this leads to us considering two or more noticeably different future states. And when this is due to us not being able to predict how a human would decide a problem, we call this a "choice".

You're assuming the Many Worlds Theory has been proven true. It hasn't. Otherwise it wouldn't be a theory.

These 'hypothetical universes' are a terrible foundation for your explanation, because they're just a bunch of unproven scenarios. You don't know if what you're purporting is really true. You just assume it is, without evidence.

Determinism states that there are no infinite possibilities. That there is no choice, only the illusion of it. That there is only one universe, where everything that has happened, is happening and will happen, has already been predetermined and every action that a being undertakes in it, furthers that one chain of cause and effect.

You have no choice because choice implies that whatever actions you are pondering are different, when they aren't. You will do what you have been predetermined to do, you just don't know what that action is until you undertake it.

If you go to the future and see yourself doing something, then go back and try your best to avoid doing it, in the end it won't matter and you will end up doing it anyway (and if you had arrived earlier in the future, you would have seen yourself flailing about, trying to avoid the inevitable). Self-fulfilling prophecies work like this as well.


Even though "choice" does not exist, it is a meaningful definition as it invokes hypotheticals that represent our lack of confidence in our predictive power.
Sure, it's only an illusion of choice as our choices are pre-determined. But that illusion exists due to our lack of ability and that lack is not going away just because we think the universe is deterministic.

From an in-character viewpoint, you're right, because they are unaware of the One True Chain of Events that has been predetermined. From an out-of-character viewpoint, you're wrong, because you, as the author, know what's going to happen, and therefore you know that there are no hypothetical alternatives and that their choices are meaningless.


Likewise, the illusion remains and is still useful both as a descriptive term and a functional one. Since we can't even *describe* the universe to infinite accuracy, or even all of it at nearly no accuracy, then referring to any description includes a huge number of hypotheticals. Describing branching into major differences due to human decisions is called a choice.

Again, that only works if you're putting yourself in the character's shoes. If you're talking about the author of the story/campaign world/etc, you're flat-out wrong because those hypotheticals don't exist. There is no branching and there is no choice.


You seem to think that the universe doesn't include human choice or knowledge. Am I reading this right?

That is correct. Such things are outside the scope of the laws of physics as we currently understand them, and therefore we cannot prove that the universe actually includes them. For all we know, they might just be the intangible delusions of our brains as they try to make sense of their surroundings and actions.

jseah
2011-11-26, 08:22 AM
That is correct. Such things are outside the scope of the laws of physics as we currently understand them, and therefore we cannot prove that the universe actually includes them. For all we know, they might just be the intangible delusions of our brains as they try to make sense of their surroundings and actions.
Ok, so that's where the problem is and why we can't understand each other. I won't reply to all the others since our disagreements / misunderstandings all stem from this.

I would disagree with this but that would not be the point of this thread. Suffice it to say that for the purposes of consistent only time travel, consistency also takes into account any and all hidden variables like choice, knowledge and intentions.

It has to. A "you" that is identical in every way but for any one of choices, knowledge and intentions is not identical.

Why would you say that human thoughts are outside the realm of physics? People suffering brain damage have taught us alot about how the brain works and which areas are responsible for what purposes.

Furthermore, various experiments (http://en.wikipedia.org/wiki/Neuroscience_of_free_will#The_Libet_experiment) have shown that people aren't even aware of when they really make a decision.

Shadowknight12
2011-11-26, 11:09 AM
Ok, so that's where the problem is and why we can't understand each other. I won't reply to all the others since our disagreements / misunderstandings all stem from this.

I would disagree with this but that would not be the point of this thread. Suffice it to say that for the purposes of consistent only time travel, consistency also takes into account any and all hidden variables like choice, knowledge and intentions.

It has to. A "you" that is identical in every way but for any one of choices, knowledge and intentions is not identical.

That makes even less sense when it comes to determinism, since now, when you go back in time, you can't even influence people's thoughts or intentions. Everything must stay *exactly* the same, right down to the immaterial. Which means that your job as a storyteller has suddenly got incredibly harder, since you have to account for everything that happened while the character(s) time-travelled and assume that everything they do was exactly what happened. If they run up and slap someone, then that should have already happened in their past. Mind you, that was what you already had to do. Now you have to take into account the preservation of the immaterial. Is X character thinking about trees? Then that's what he thought before the time-travel, and everything the characters do must further that state.

And no, choices have an observable effect because they translate into actions, so different choices would cause an observable difference.

A "you" that is identical in every way but knowledge and intentions is still identically you, as far as physics/the universe can tell. As long as you don't act on that knowledge/intentions and you act exactly the same, then the universe will be none the wiser.

At most, you can argue that a difference in mood correlates with a difference in the subject's neurochemistry, but since determinism is at play, then the mood will be preserved because the same neurotransmitters must be released at their predetermined times.


Why would you say that human thoughts are outside the realm of physics?

Because they are. They are not part of the physical world, and there's plenty of evidence that shows that they are not affected by physical laws. Have you ever seen a thought with mass? If it's energy, can it be measured? No. They are outside the scope of how we currently understand physics.


People suffering brain damage have taught us alot about how the brain works and which areas are responsible for what purposes.

Furthermore, various experiments (http://en.wikipedia.org/wiki/Neuroscience_of_free_will#The_Libet_experiment) have shown that people aren't even aware of when they really make a decision.

What you're not quite getting is that none of this proves anything about knowledge itself. Any experiments conducted on a brain can easily be argued to apply only to the subject's perceptions. If I raise a wall between someone and a flower, I am affecting their perceptions of the world. So if I conduct a trepanation and cause massive amnesia, you can't prove I have actually affected knowledge in any way. I may have merely affected the subject's ability to perceive or recall it.

So for better or worse, if we can't observe nor interact with knowledge, it's effectively outside the scope of current science. Perhaps, in the future, we may invent techniques that allow us to directly observe and interact with knowledge itself. Or we may not.

Rogue Shadows
2011-11-26, 11:13 AM
This is time travel of the consistent type (http://en.wikipedia.org/wiki/Novikov_self-consistency_principle). Aka, the most confusing to use. When you time travel backwards, there will exist two versions of you. Changing the past is not possible, because if you did change the past, it would not have been what it was before you traveled. (inconsistent time loops don't occur)

This is unfun. What's the point of time travel if you can't do anything? Yes, yes, yes, all sorts of important discoveries about people, places, and things, but that doesn't make for a very compelling game.

Also, I'm pretty sure, impossible, if time travel is possible, unless every instance of time travel is nothing more than a stable time loop - i.e., causing something to happen in the past because that's how the event in the past always went down, with you causing it - and stable time loops bring us back to "this is unfun."

If I ever time traveled, and found the universe to be deterministic, the first, last, and only thing I would do is dedicate the remainder of my existance to trying to kill me in the past, and screw the Universe if it creates a paradox that destroys it.

Yes, I would eliminate all life throughout all time just to make a point. And the point would be: determinism is stupid, since it pretty much eliminates any delusions of free will. I much prefer a chaotic, unpredictable universe, or at the very least for the universe to neglect to inform me in such blunt terms that it is deterministic.

No, I much prefer proper, fun time travel. Wibbly-wobbly, timey-wimey...stuff.

Also I think I'm the first person in history to consider the full implications of a) time travel is possible and b) the past can be changed.

Think about every time travel story you have ever read or watched or heard where the past can be changed. They all have one thing in common that makes no sense. Pop quiz: Can you guess what it is?

vegetalss4
2011-11-26, 11:43 AM
Think about every time travel story you have ever read or watched or heard where the past can be changed. They all have one thing in common that makes no sense. Pop quiz: Can you guess what it is?

Nope, I can't think of something, unless you mean causeless effects, such as a time traveller who succeeds in changing the past would remove his reason to do so, but that don't appear in or apply to all relevant stories I have heard.

Douglas
2011-11-26, 12:06 PM
Because they are. They are not part of the physical world, and there's plenty of evidence that shows that they are not affected by physical laws. Have you ever seen a thought with mass? If it's energy, can it be measured? No. They are outside the scope of how we currently understand physics.
Um. What.

This is much the same as saying that the data on your computer's hard drive is "not part of the physical world". Have you ever seen a megabyte with mass?

It is an argument that is completely beside the point and misses the very nature of knowledge and how it is stored. The physics of knowledge have nothing whatsoever to do with mass and everything to do with how certain pieces of matter are arranged. It is true that we lack comprehensive detailed knowledge of how to decipher the meaning of any particular arrangement in the human brain, but that is a far cry from saying we don't know that knowledge is stored in such arrangements.

As for the free will vs determinism thing, I'll just leave a few quotes taken from this (http://www.mit.edu/people/dpolicar/writing/prose/text/godTaoist.html):

The word determined here is subtly but powerfully misleading and has contributed so much to the confusions of the free will versus determinism controversies. Your acts are certainly in accordance with the laws of nature, but to say they are determined by the laws of nature creates a totally misleading psychological image which is that your will could somehow be in conflict with the laws of nature and that the latter is somehow more powerful than you, and could "determine" your acts whether you liked it or not. But it is simply impossible for your will to ever conflict with natural law. You and natural law are really one and the same.

Don't you see that the so-called "laws of nature" are nothing more than a description of how in fact you and other beings do act? They are merely a description of how you act, not a prescription of of how you should act, not a power or force which compels or determines your acts. To be valid a law of nature must take into account how in fact you do act, or, if you like, how you choose to act.

Rogue Shadows
2011-11-26, 12:10 PM
Nope, I can't think of something, unless you mean causeless effects, such as a time traveller who succeeds in changing the past would remove his reason to do so, but that don't appear in or apply to all relevant stories I have heard.

Nope. If the past can be changed at all, then causless effect must be possible. Presumably Time compensates somehow.

No, the problem is the basic assumption that there is only one time machine and/or that only one point in the past is ever traveled to and has its history changed.

Here's a simple version of how it works:

1) I invent a watch-mounted time machine, intending to kill Hitler.
2) I travel to the past to 1933 Germany.
3) Germany has not even united as a nation yet. Russia is a backwards, agrarian nation run by the Czars still. Italy is a major world power de facto as well as de jure. There is an Empire of California.
4) Confused, I return to my present.
5) In the present, the Soviet Union controls most of the world, with America a collapsing state ravaged by internal dissent. Australia is the last true bastion of democracy. Pogroms have killed off virtually the entire Jewish culture.
6) Horrified, I race back into my time machine and go back to 1917, intent on stopping Lenin.
7) Russia doesn't exist as a single united state, and large swaths of it are governed by China and the Ottoman Empire. The most common language in the world is Spanish, since the Spanish Empire never fully fell.
8) I try again to return to my own time, but once again find it utterly unrecognizeable.

Basically: if time travel is possible, and it is possible to change the past, then it is impossible to travel back to the past as you recognize it, or the future as you recognize it, because you're hardly going to be the only person ever to invent a time machine and try to change the past. Further, traveling in time for even one second either way removes you from the timeline, so by definition you'll still end up in an unrecognizeable world.

Travelling to the future has an added drawback: you are travelling to a future where you at one point in the past disappeared without a trace.

Let's back up for a moment and look at things in detail. Say I invented a watch-mounted time machine and successfully ended up in the 1933 Germany I had intended to, and I kill Hitler.

Nazi Germany never rises. The timeline is changed. In this new timeline, someone else is going to eventually invent a time machine. Let's say it's a guy from Soviet Union of 2017, which remains a world power. Sick of the fact that the USSR still hasn't won the cold war against America, he travels back in time (with his handy wrist-mounted time machine) to 1898 and kills Theodore Roosevelt.

For the purposes of our discussion, these are the only two instances of time travel that ever happen, ever. Because of the way time and space happen, however, as far as Time is concerned, these two events happened simultaneously, just at different points in the timeline.

So, upon killing Hitler (and our loyal Soviet worker killing Roosevelt) simultaneously, when I go back to my present, I am going back to a world where Roosevelt never became President, and there was never a Hitler running Germany. Obviously, this future, my present, is going to be bizarrely different from the present that I left behind.

Now, it would seem that our loyal Soviet Worker has just about a dream to go back to: he is not only from a timeline where the Nazis never rose (he doesn't even know what one is), but he is also from one where, now, America lost one of its greatest Presidents. Surely he will return to a worker's paradise!

But the problem is that we specifically eliminted anything from occuring in his timeline. If we remove that, then his killing of Roosevelt will also create a new timeline. Say that this results in a South African in 1967 invented a time machine and travelling back to 1067 with advanced technology and proclaiming himself Supreme Ruler of Europe. Of course, that creates another timeline where someone from the High Chechan State in 2155 goes back a hundred years provides future knowledge to his ancestors so that he can live it big. Then another...and another...

...and even this is simplified because it presumes that each time a new timeline is created, only one person has a chance to travel back in time in it. But then again, all time is simultaneous. So in reality what we have is X number of time travellers from Y timelines always travelling backwards in time to change time, resulting in more timelines and more time travellers travelling backwards in time to change time, over and over and over again...

And it is important here to note that "timeline" is innacuate, since modern contrivance has it that a "timeline" represents a branch from the "main" flow of time. That isn't happening here. What's happening is that the flow itself is changing, again and again, over and over and over, all simultaneously, infinitely...

...so, yeah. In sum: It is impossible to travel to "your" past. Having travelled to "a" past, it is impossible to travel back to "your" present. And every time you time travel at all, you end up in a completely different timeline.

Have fun thinking about that.

Douglas
2011-11-26, 12:15 PM
Nope. If the past can be changed at all, then causless effect must be possible. Presumably Time compensates somehow.

No, the problem is the basic assumption that there is only one time machine and/or that only one point in the past is ever traveled to and has its history changed.
How to end time travel (http://www.smbc-comics.com/index.php?db=comics&id=1887#comic).:smalltongue:

Rogue Shadows
2011-11-26, 12:21 PM
How to end time travel (http://www.smbc-comics.com/index.php?db=comics&id=1887#comic).:smalltongue:

Well...no. That scenario assumes that all time travellers arrive at the same point in time and space. What really happens is some arrive early, some arrive late, some arrive right in the party, some arrive elsewhere and just make their way there...

Plus all that needs to really happen is for two time travellers to try and arrive at the party in the same place and time. As I understand it, they will explode. No more party. Timeline instantly changes throughout all of time for there to have been a massive explosion in that location where a party was being held.

Shadowknight12
2011-11-26, 12:26 PM
Um. What.

This is much the same as saying that the data on your computer's hard drive is "not part of the physical world". Have you ever seen a megabyte with mass?

Data is a way we have to identify information. This information can be traced back to a series of ones and zeroes physically engraved on some concrete source. Data itself is outside the laws of physics and is not part of the physical world. The source of that data (the series of engraved ones and zeroes) are part of the physical world and are therefore affected by its laws.

The problem with data and information is that they possess an extra meaning that we assign to them that misleads people into thinking they are more than an engraving, plus electricity pulses along a circuit and light being emitted from a monitor. Those are all physical things that are governed by the laws of physics.

The symbols that they display are completely irrelevant, because they have no meaning of their own. We assign meaning to them and designate them 'data,' but all the baggage that comes with that world is extraneous to the physical universe, and exists only in our mind.


It is an argument that is completely beside the point and misses the very nature of knowledge and how it is stored. The physics of knowledge have nothing whatsoever to do with mass and everything to do with how certain pieces of matter are arranged. It is true that we lack comprehensive detailed knowledge of how to decipher the meaning of any particular arrangement in the human brain, but that is a far cry from saying we don't know that knowledge is stored in such arrangements.

And the arrangement of certain pieces of matter is only given sense by us, and this sense that we are giving them exists purely in our minds and not in the physical world.

Douglas
2011-11-26, 12:37 PM
And the arrangement of certain pieces of matter is only given sense by us, and this sense that we are giving them exists purely in our minds and not in the physical world.
And both the "us" that gives sense to it and the mechanism by which we do so are composed of physical pieces of matter and electricity and other physical things, all of which are in the physical world and subject to physical laws. The arrangement that stores the knowledge, the organic device that reads the knowledge, the organic computer that assigns meaning to it, etc. are all - every single one - physical things, and the sum total of their combined operation is thus also physical.

The only way to get around this is to bring something like souls into it (and even that only does it with certain non-universally-accepted ideas of what souls are and how they work), in which case we'll have to agree to disagree, and stop this avenue of discussion because that route unavoidably heads into religion.

Shadowknight12
2011-11-26, 12:47 PM
And both the "us" that gives sense to it and the mechanism by which we do so are composed of physical pieces of matter and electricity and other physical things, all of which are in the physical world and subject to physical laws. The arrangement that stores the knowledge, the organic device that reads the knowledge, the organic computer that assigns meaning to it, etc. are all - every single one - physical things, and the sum total of their combined operation is thus also physical.

Yes, but the definition of the words "data" and "information" imply that there is a meaning beyond the physical. If the arrangement isn't performed obeying certain systems and classifications, it's not information, and therefore it can't be data. This extra quality that transcends the physical is that which is outside the laws of physics by its very definition. Until we can find a way to observe, physically influence or measure meaning, data and all things immaterial will remain outside the scope of physics.


The only way to get around this is to bring something like souls into it (and even that only does it with certain non-universally-accepted ideas of what souls are and how they work), in which case we'll have to agree to disagree, and stop this avenue of discussion because that route unavoidably heads into religion.

What? You completely lost me. What have souls got to do with anything?

vegetalss4
2011-11-26, 01:29 PM
Have fun thinking about that.

Your theory has a few flaws.

before I start let us agree on some terminology. The points in time before the one you currently are in (i.e. the past) as downline, and those after said point (i.e the future) as upline, both combined are the timeline.
The past from your point of view will be referenced to as your Yester
and your personal future is your Yet, both capitalised.
Lets use time traveller to anything which travels through time, whether living or not, and a facilitator to refer to someone or something which can make something (possible themselves) travel through time, either a time machine or someone with the inherent ability.
This will make the topic so much easier to discuss.

first note that the point on the timeline which is the present changes whenever anybody uses a facilitator, also note that I am assuming that anyone outside of the timeline is protected from changes to the timeline.

In a universe where one can change the downline has, by definition, a non-defined upline i.e, the future can be changed too.
As such any time machines/natural time travellers which do not exist at the present, do not have to be considered.
Those in the downline because they have already (In your Yester) made the changes they are going to make [1](from their point of view) thus forming your Yester, and then either travelled upline in relation to you, or died.
Those who are upline don't have to be considered since your actions in the present will decide their Yester and thus effect if they even are going to be able to travel in time and which changes they are going to want to make.

Now, whenever you travel to the past anyone else who can travel in time in your old present are now upline from you and as such stop existing[2] until the timeline is allowed to move back up to their present at which point their Yester will have been affected by any changes you make downline.
This means that they now (as upline time travellers) are irrelevant for your perception.

As such there is only one circumstance where there is more than one time traveller at once, namely if all of them travelled from the exact same point in time to the exact same point in time[3] with no rounding.

The result is that whenever you travel downline you arrive at the downline of your former present[4], then you change whatever you was going to change, which may at some point cause someone upline to go back and change the downline some more, but if they do they will also change your Yester and as such you won't notice.

Now travelling upline is much more dangerous, because not only will you appear in a future where you suddenly disappeared, but if in the period of the timeline you travelled past includes any facilitators then any time travels that start then will also change the future you arrive in, including possibly making it so that you never time travels and thus can meet the you who didn't travel.

note that if you ever travel past a point with a facilitator, and it is used, it then becomes pretty much impossible for you to arrive at your original present ever again.

[1] not withstanding any changes you make to their choices in your Yet of course.
[2]instantly, such that it doesn't even take them a single point in time.
[3] I am here referring to literal indivisible points in time which may not even exist, if such don't exist then it is only possible by using the same facilitator.
[4] which will fit your Yester if this is the first time you time travel.

Douglas
2011-11-26, 01:31 PM
Yes, but the definition of the words "data" and "information" imply that there is a meaning beyond the physical. If the arrangement isn't performed obeying certain systems and classifications, it's not information, and therefore it can't be data. This extra quality that transcends the physical is that which is outside the laws of physics by its very definition. Until we can find a way to observe, physically influence or measure meaning, data and all things immaterial will remain outside the scope of physics.
...

So, something that is defined by a physical arrangement, interpreted by a physical device, given meaning by a complex series of physical interactions, and is the result of a combination of exclusively physical things... is not physical?:smallconfused:

Every single part of what makes knowledge or data what it is is physical, and the combination of physical things cannot produce a non-physical result. If you change the arrangement of a brain, you have changed the knowledge stored within it. If the arrangement stays the same, then so does the knowledge.


What? You completely lost me. What have souls got to do with anything?
They're a potential non-physical source of meaning and decisions, and such a source is necessary for knowledge, minds, and decisions to not be physical (if incredibly complex) things.

Shadowknight12
2011-11-26, 01:44 PM
...

So, something that is defined by a physical arrangement, interpreted by a physical device, given meaning by a complex series of physical interactions, and is the result of a combination of exclusively physical things... is not physical?:smallconfused:

That is correct. Because data is more than the combination of the physical objects that compose it. Data has a meaning that humans assign to it, and without that meaning, it cannot be data. That meaning is intangible and is not a part of the physical world, and therefore neither is data, for without that meaning, data would be nothing but a random assortment of physical events.


Every single part of what makes knowledge or data what it is is physical, and the combination of physical things cannot produce a non-physical result. If you change the arrangement of a brain, you have changed the knowledge stored within it. If the arrangement stays the same, then so does the knowledge.

That is wrong. There is a part of what makes data what it is, and that part is meaning. As I explained above, meaning is immaterial, and therefore unaffected by the laws of physics. Anything that stems from physical sources (like data or emotions) but requires an immaterial component to be what it is, cannot be affected by the laws of physics. That is not to say that their sources (the physical aspects of which they stem) are not affected by the laws of physics, because they are.

As for knowledge, that one's even easier, because knowledge itself is a purely mental construction. At least data has a firmly-rooted source in the physical. How can you prove, what evidence can you produce, that knowledge in itself is not purely immaterial?

And no, there is no proof that by changing the rearrangement of a brain, you're changing actual knowledge. As I said twice already, it's just as likely that you're changing the subject's ability to perceive or recall that knowledge.


They're a potential non-physical source of meaning and decisions, and such a source is necessary for knowledge, minds, and decisions to not be physical (if incredibly complex) things.

Potential, not necessary. And completely irrelevant to this discussion.

vegetalss4
2011-11-26, 02:09 PM
That is correct. Because data is more than the combination of the physical objects that compose it. Data has a meaning that humans assign to it, and without that meaning, it cannot be data. That meaning is intangible and is not a part of the physical world, and therefore neither is data, for without that meaning, data would be nothing but a random assortment of physical events.


To illustrate this with an example: (hope you don't mind:smallsmile:)

lets take Williams Shakespeare's Hamlet as our piece of data in question.
Now you can find Hamlet recorded on many different physical objects, written in books, on computers as recording of different performances of it, ect. ect.

Now all the different physical objects have in common that they contain Hamlet. But no matter which physical phenomena one where to effect the objects with, Williams Shakespeare's Hamlet would not be affected in the slights, as (I hope) is self-evident.
the same thing holds true for all data and knowledge, and thus since no physical phenomena can affect them, including the physical laws, they themselves are not physical.

jseah
2011-11-26, 02:21 PM
Shadowknight12:

That makes even less sense when it comes to determinism, since now, when you go back in time, you can't even influence people's thoughts or intentions. Everything must stay *exactly* the same, right down to the immaterial.
<...>
Is X character thinking about trees? Then that's what he thought before the time-travel, and everything the characters do must further that state.
Correct. That is the level of consistency that I mean when I say consistent time loops only.


A "you" that is identical in every way but knowledge and intentions is still identically you, as far as physics/the universe can tell.
Not to the consistency principle I am invoking for this time travel thought experiment.

If it has any effect at all, even if its only on thinking and "invisible" things like knowledge, then it will be consistent.

Perhaps I shall define universe for the purpose of this time travel.
Universe is a term that includes all things, material and immaterial, that can have an effect on other things, whether they actually have an effect or not; their current states, both their properties and states relative to each other, are also part of the universe.
If something can have *any* effect at all, no matter how small or how unnoticeable or only under unique circumstances, it is part of the universe and thus is deterministic and subject to the consistency principle.

And so therefore, under this definition of the "universe":

And no, choices have an observable effect because they translate into actions, so different choices would cause an observable difference
choices are part of the universe as it has an effect.

The knowledge that underlies those choices affect choices. Since choices are part of the universe, the knowledge is as well.

If they are all part of the universe, they are all likewise deterministic and must be consistent in a time loop.
For the purposes of discussing the hypothetical time travel I lay out in the OP, this is what I mean by consistent.


What you're not quite getting is that none of this proves anything about knowledge itself. Any experiments conducted on a brain can easily be argued to apply only to the subject's perceptions.
Under a certain philosophy, namely one that I subscribe to but don't know the name of, humans don't have knowledge as you define it. We only perceive things. (as you define perception)

In this philosophy, we are nothing more than our brains and bodies. Data is represented *only* by its arrangment in physical media and without that physical representation, there is no data.
In fact, "data" or "knowledge" is a simplification, and not a real thing. You use it when you refer to the physical representation because its easier.


Also, I'm pretty sure, impossible, if time travel is possible, unless every instance of time travel is nothing more than a stable time loop - i.e., causing something to happen in the past because that's how the event in the past always went down, with you causing it - and stable time loops bring us back to "this is unfun."
Well, this form of time travel makes the time traveller more like a 4-dimensional being.

Coming up with stable loops can be hard, but the challenge of doing so and finding a *useful* stable time loop is what I find fun about this type of time travel.
Not to mention, the baffled looks I get when I describe a particularly complex loop (eg. one-atom key) is worth all the trouble. :smallamused:

Its about the challenge and sheer insanity of thinking in 4 dimensions.

Wrt "creating a paradox to destroy the universe":
I can rest assured that, if you do discover time travel and it turns out to be this consistent type, you will find creating a paradox impossible. That's part of the initial assumptions; that it is not possible to create a situation that has no solutions.

Erfworld reference:
Don't fight Fate, embrace it and your life will be easier.

Translation:
Don't bother engineering paradoxes for the universe to solve. The solutions are unlikely to be productive.
Instead engineer *almost* paradox situations such that their solutions give you an effect you want.

jseah
2011-11-26, 02:34 PM
To illustrate this with an example: (hope you don't mind:smallsmile:)

lets take Williams Shakespeare's Hamlet as our piece of data in question.
Now you can find Hamlet recorded on many different physical objects, written in books, on computers as recording of different performances of it, ect. ect.

Now all the different physical objects have in common that they contain Hamlet. But no matter which physical phenomena one where to effect the objects with, Williams Shakespeare's Hamlet would not be affected in the slights, as (I hope) is self-evident.
the same thing holds true for all data and knowledge, and thus since no physical phenomena can affect them, including the physical laws, they themselves are not physical.
I will reference Hume's Fork (http://en.wikipedia.org/wiki/Hume's_fork) here. Specifically this bit:


Third, Hume notes that relations of ideas can be used only to prove other relations of ideas, and mean nothing outside of the context of how they relate to each other, and therefore tell us nothing about the world. <...>
... logical statements ... are always true and unchanging, Hume held that, while true, they contain no formal reality, because the truth of the statements rests on the definitions of the words involved, and not on actual things in the world ...

In this case, Hamlet is defined as the text of Hamlet. If I have a thumbdrive that has Hamlet on it, for my statement to be true, my drive has to have a representation that we agree is representative of the text of Hamlet.
In Hume's words, what you say is "Hamlet" is a statement about ideas.

If I damage my drive and the first two words of the Hamlet text is deleted, then what is on my drive is not Hamlet anymore.
My thumbdrive not having a representation of the idea of Hamlet is a statement about the world.

In any case, the definition of Hamlet has absolutely no effect on how my thumbdrive operates.


By his own words, Shadowknight12 says that choices have an effect on the world. Therefore, however he thinks human knowledge operates, it becomes a "statement about the world".
For the purposes of this hypothetical time travel, that must also be deterministic and consistent. Because as I have defined how the time travel works, all "statements about the world" must be consistent throughout the time loop.

Rogue Shadows
2011-11-26, 02:42 PM
Don't bother engineering paradoxes for the universe to solve. The solutions are unlikely to be productive.
Instead engineer *almost* paradox situations such that their solutions give you an effect you want.

Trouble is, in the event of a deterministic universe, the effect I want is "destroy the universe out of spite."


Its about the challenge and sheer insanity of thinking in 4 dimensions

It's easier than you think. Or at least I've never found it particularly hard...

Mind, I like the idea of a stable time loop; I just want the universe to be such that it's possible to kick a stable time loop, unstabalize it, but the universe continues anyway, because Time is made of sterner stuff than it's given credit for.

I especially like universes were paradoxes are actually impossible because Time hates you. That is, if you travel back in time and kill your own grandfather at age 10, he dies (because you killed him), you pop out of existance (because Time hates you), but your father is fine (because he didn't do anything wrong).

Also I like the idea of "entrenched" time; that is, time ends up flowing along the same course eventually, but it's still possible to change things.

So say I go back in time and kill Hitler; there's no Nazi Germany and perhaps no WWII (we don't necessarily end up in a Red Alert scenario). However, the moon landing still occurs on or around the year 1969.

Basically, my ideal take on Time is that Time is an infinitely wide but finitely deep river. Toss in a rock, and Time now has to flow around that rock. But eventually Time finished flowing around the rock and resumes along its normal course, albeit with now a few pieces of the rock having eroded off and coming along for the ride.

jseah
2011-11-26, 02:59 PM
It's easier than you think. Or at least I've never found it particularly hard...
Perhaps you could contribute a more complex loop than these?

Post 50 (http://www.giantitp.com/forums/showpost.php?p=12275303&postcount=50)

As well as the few examples I gave in the OP.

Rogue Shadows
2011-11-26, 03:27 PM
Perhaps you could contribute a more complex loop than these?

Dunno if I can contribute; your terms are remarkably technical, while I've always found time travel easier to both express and understand if you stick to fairly casual language.

Also, nothing you're outlining is very "complex." They seem like the straightforward results of a deterministic universe and time travel. They also seem to all just be expressions of a basic ontological loop after you described that; i.e., I don't see a real difference between a basic ontological loop and interlocking ontological loops. That's just two (or more) loops that intersect and not really worth defining on its own: when a cup is placed on top of a saucer we don't come up with a new term for the set or attempt to redefine what they are.

Similarly, a material non-ontological loop is just an ontological loop is only non-ontological for you, the time traveller, but it's still fundamentally the same thing for whatever material is being shunted around through loops. Perspective has changed; the event has not.

I dunno, maybe I played Time Splitters 3 too soon after playing Portal.

Although I'm going to point out that your entry for "non-time travel (default)" is technically incorrect. We all travel through time at a rate of one second experienced per one second travelled. "Non-time travel" is an impossibility: something that doesn't time-travel pops into existance from nothing and then in the same instance ceases to exist.

I'm not sure how the universe would reconcile such an event occurring. Most likely it'd be a Big Bang. Fortunately I'm given to understand that the nascent cosmos so created would immediately be folded into its own brane and cease to interact with our own, and so we'd be safe.

jseah
2011-11-26, 04:36 PM
The difference between basic and interlocking is that when you nest basic loops inside each other, each new loop is only dependent on itself and maybe the loops they are nested in. Loops are not dependent on loops inside themselves.

Interlocking loops are dependent on each other. They either all happen, or they don't at all.

In basic, time travel event A enables event A. While in the loop of enabling A, time travel event B is nested inside. Time travel event B enables B.
Eg. the key to door A is behind door A. But the key to door B is behind door A and door B. Using 2 ontological loops to open the doors is nesting a basic ontological loop inside another.
A enables A. A + B enables B.

You can treat them as distinct and separate events.
You could just as easily have only opened door A and left door B unopened.


In interlocking loops, the key to door B is behind door A. The key to door A is behind door B. To open both doors, you need 2 ontological loops.
A enables B. B enables A.

In this case, the loops depend on each other and are NOT distinct and separate events.
You cannot open door A and not open door B.

EDIT: I drew some diagrams (http://dl.dropbox.com/u/10120644/Time%20Travel%20ontological%20loops.png).


The reason why I highlight this difference comes from my short experience in Achron. Achron uses a version of consistent time travel for the final timeline and the game is about influencing which particular consistent loops happen so that it resolves in your favour.
Of course, paradoxes can be created and in Achron, this resolves into timelines that flick between one or another state, both of which are inconsistent. This is mainly due to limitations in a game format which prevent proper consistent resolution and so Achron's time travel is NOT a proper consistent time travel.

The difference between the two ontological loops (basic and interlocking) is that if you have an ontological loop set up somewhere and your opponent pokes it, the basic version is more stable and harder to disrupt.
In basic, if your opponent disrupts loop B, then door B doesn't get opened.
In interlocking, if your opponent disrupts either loop, then the entire thing comes down.

Its only important when there are multiple time travellers that aren't cooperating.

------------------------------------------------------------

I admit, I ran out of words to describe the material non-ontological loop vs material ontological loop. Perhaps you could think of a better way to say it?

What I meant was that in non-ontological, the "borrowed" material comes from somewhere else in the universe and exits in another place.
Eg. "borrowing" a key from the future to open a door to get at that key. or "borrowing" a few atoms a trillion times over to make a key.

In material ontological loop, the "borrowed" material comes from itself.
Eg. "borrowing" a key from the future to open a door, and giving that same "borrowed" key to your past self.

If you could uniquely label every particle in the universe, you would see two copies of the particles that make up your "borrowed" key in the material non-ontological loop.
In the material ontological loop, there are now *more* unique particles in the universe than before the loop started. These extra unique particles go away when the loop ends.

The functional difference is that the material ontological loop is incredibly hard to set up since thermodynamics indicates that making a consistent one is incredibly difficult. The state of your key is extremely unlikely to become the same as the key that left.
(or at least same material state. The particles can be shuffled around a bit as long as they all exchange positions nicely. There could be less unique particles than the number of particles in the "borrowed" object since some of them could loop multiple times.
In fact, I submit that this is what will almost certainly happen since the number of combinations that are allowed by this is vastly larger than those where every particle returns to its original position and energy. )

"borrowing" one atom and then "returning" it is simple. But doing that for the ~2*10^22 atoms involved in a pure iron key of around 11 grams gets ridiculously unlikely.

Shadowknight12
2011-11-26, 05:30 PM
To illustrate this with an example: (hope you don't mind:smallsmile:)

Not at all!


Perhaps I shall define universe for the purpose of this time travel.
Universe is a term that includes all things, material and immaterial, that can have an effect on other things, whether they actually have an effect or not; their current states, both their properties and states relative to each other, are also part of the universe.
If something can have *any* effect at all, no matter how small or how unnoticeable or only under unique circumstances, it is part of the universe and thus is deterministic and subject to the consistency principle.

Then you have to admit that the laws of the universe don't apply to the entirety of the things you call universe. See my previous point re: immaterial things not being subject to the laws of physics as we currently know them. If you're fine with that, knock yourself out.


choices are part of the universe as it has an effect.

I never said they didn't. I said they did.


The knowledge that underlies those choices affect choices. Since choices are part of the universe, the knowledge is as well.

If they are all part of the universe, they are all likewise deterministic and must be consistent in a time loop.
For the purposes of discussing the hypothetical time travel I lay out in the OP, this is what I mean by consistent.

Like I said above, accepting that means accepting that there are things in the universe that do not conform to its laws. That opens up a huge can of worms but it's ultimately yours to deal with.


Under a certain philosophy, namely one that I subscribe to but don't know the name of, humans don't have knowledge as you define it. We only perceive things. (as you define perception)

But then how do you explain memory? You're not perceiving anything when you make a recollection. Or dreams? What are you perceiving when you dream? And what about thoughts? Can you influence thoughts via a purely physical mean (e.g., the right combination of neurotransmitters and electrical discharges will get someone to think about dogs)?


In this philosophy, we are nothing more than our brains and bodies. Data is represented *only* by its arrangment in physical media and without that physical representation, there is no data.
In fact, "data" or "knowledge" is a simplification, and not a real thing. You use it when you refer to the physical representation because its easier.

So you claim that no immaterial things exist? Then what are thoughts, data and knowledge, then? Where are they located, physically? Not their sources, mind you. We all know that brains and books may store knowledge, but they aren't knowledge itself. Can you influence such things physically? If so, how? Are they ruled by the Law of Conservation of Matter or Energy? I mean, if they exist, physically, they must come from somewhere and go somewhere after they've expired. And if they're either energy or matter, then the Law of Conservation must apply, which means that there's a cycle of transformation for thoughts, knowledge and the like. Where is it? How does it work?

Do explain, I'm terribly interested in hearing how such a philosophy works.

jseah
2011-11-26, 05:38 PM
So you claim that no immaterial things exist?

Then what are thoughts, data and knowledge, then? Where are they located, physically?
<...>
Do explain, I'm terribly interested in hearing how such a philosophy works.
Yes, the philosophy is that those things don't exist.

I haven't replied to the earlier stuff since this negates all of that.

Choice being part of the universe is by *your* definition.


But then how do you explain memory? You're not perceiving anything when you make a recollection. <...>
Can you influence thoughts via a purely physical mean (e.g., the right combination of neurotransmitters and electrical discharges will get someone to think about dogs)?
You ARE perceiving something when you make a recollection. That thing is your brain. More precisely, when you think of something, your thoughts are really just chemical and electrical signals between cells.

And yes, theoretically possible to "mind control" people by pushing all the right buttons. There'll be one heck of alot of buttons to push in a very complicated pattern.

As shown in my link earlier, researchers were able to make people choose which hand to move, albeit unreliably and under controlled conditions. We're still at the very crude stage, and it might never be feasible, but I believe that such a thing is theoretically possible.

Shadowknight12
2011-11-26, 05:48 PM
You ARE perceiving something when you make a recollection. That thing is your brain. More precisely, when you think of something, your thoughts are really just chemical and electrical signals between cells.

And yes, theoretically possible to "mind control" people by pushing all the right buttons. There'll be one heck of alot of buttons to push in a very complicated pattern.

As shown in my link earlier, researchers were able to make people choose which hand to move, albeit unreliably and under controlled conditions. We're still at the very crude stage, and it might never be feasible, but I believe that such a thing is theoretically possible.

See, what makes no sense is how perceiving a select part of your brain can trigger a memory. What does it mean? That memories are 'stored' somewhere in your brain, like what Assassin's Creed would have you believe to justify its plot? And if they are stored there, then what are they made of, physically? And if they aren't stored anywhere, then how can you access a specific memory by perceiving your brain? Is your brain like a hard-drive that engraves something upon itself whenever you experience something? And if it does, then what engraving system is it using, since it's capable of storing such an incredible amount of detail? And how can it keep storing memories without running out of engraving space? It must have the best encryption system ever.

Are you aware that if thoughts were merely chemical and electrical signals between cells, then a lot of 'brain dead' cases wouldn't exist? Because there's plenty of people who are very much brain dead and still have perfectly normal neurobiochemistry and electrical conduction.

That should prove that there's more to thoughts than that, and that perhaps neuroscience is too complex and full of unknowns to make simplistic assumptions.

jseah
2011-11-26, 06:21 PM
That should prove that there's more to thoughts than that, and that perhaps neuroscience is too complex and full of unknowns to make simplistic assumptions.
Ok, that does it. I'm aiming to do a PhD in a neuroscience related area and from the roughly 40 research papers I have read on the subject of neurotransmitter receptors quite firmly concludes that there is nothing mysterious going on in the brain.

Neurons interact with each other by more than signal propagation. There is growth signals, signals that stimulate large areas, signals that remodel or remove or add connections, etc.

The interactions between those signals and how those process and store information is the essentially the field of neuroscience.
This is so insanely complex that I doubt we will be done any time before I'm in the ground.

At the same time, the complexity comes from the interactions and emergent phenomena that arise from them. Not from some mystery... thing.


See, what makes no sense is how perceiving a select part of your brain can trigger a memory.
<...>
Is your brain like a hard-drive that engraves something upon itself whenever you experience something? And if it does, then what engraving system is it using, since it's capable of storing such an incredible amount of detail?
Memory is almost certainly in the connections that have formed between neurons. There is more than one type of connection and different types behave differently.
Neurons can connect more than once to other neurons.

There are also more than one type of neuron and they behave differently to the same connection type and make yet other connections and connect in different ways.

Let's do a very simplified, and almost certainly wrong, estimate.

You have 100 neurons in a dish. Each neuron can form (or not form) connections to 99 other neurons. Each neuron can make 10 types of connections.
Each neuron can differentiate between having one to twenty connections of the same type (it can sense how many connections it has).

One neuron has 99 neurons x 10 connection types x 20 connection strengths = 19.8 k connections

Assuming the presence or absence of a connection corresponds to one bit (1 and 0 respectively), this means that a dish of 100 neurons can store 19.8MB of information.

Assuming there are 200 neuron types in the brain (this is pretty darn conservative since a single one of the six layers of neurons in the cortex has above 20 distinct types of neurons, differentiated by cell surface receptors), that means that the same 100 neurons can exist in 200^100 = 1.27 x 10^230 different combinations of neuron types.

Sure, each specific dish combination only stores 19.8 MB, but that same connection pattern means different things if translated to a different set of neuron types.

Now, how many billion neurons do we have in our brains? Wikipedia tells me we have 100 billion...
Simply scaling up the estimate (assuming that each neuron can contact up to 100 others) puts the estimated information capacity of a human brain at 20 petabytes.

(and don't get me started on the glial cells, which outnumber the neurons and do play some sort of role. We haven't pinned it down exactly. )


Of course, that contains so many simplifying assumptions to be basically pointless. But I hope it was demonstrative of the power of combinatorial problems.

Is it any wonder that we haven't deciphered it yet? No, not at all surprising.


Are you aware that if thoughts were merely chemical and electrical signals between cells, then a lot of 'brain dead' cases wouldn't exist? Because there's plenty of people who are very much brain dead and still have perfectly normal neurobiochemistry and electrical conduction.
Take your computer chipboard. Cut a few random conduction paths (that's those silver lines)
Run wires from random conduction paths to other random conduction paths.
Then cut a (small) hole out of the chipboard, taking care to remove at least one small chip.

Does your computer still work? My guess is no.
Does your computer still have electricity running through it? Do all the bits still work the way those bits are supposed to work? Yes, if you turn it on.

Brain dead is the same deal.

Just because its made of the same stuff and individual bits still exist, does not mean that if you take away some or disrupt the connections that the thing works in the same way.

Shadowknight12
2011-11-26, 06:54 PM
Ok, that does it. I'm aiming to do a PhD in a neuroscience related area and from the roughly 40 research papers I have read on the subject of neurotransmitter receptors quite firmly concludes that there is nothing mysterious going on in the brain.

Neurons interact with each other by more than signal propagation. There is growth signals, signals that stimulate large areas, signals that remodel or remove or add connections, etc.

The interactions between those signals and how those process and store information is the essentially the field of neuroscience.
This is so insanely complex that I doubt we will be done any time before I'm in the ground.

At the same time, the complexity comes from the interactions and emergent phenomena that arise from them. Not from some mystery... thing.

I know, I've had to study that very sort of thing before, and I'm sure I'm likely to learn even more by the time I'm done with my studies.

I never said, however, that complexity arises from 'some mystery thing.' I said that there is more to neuroscience that mere electricity and biochemistry, as neuroscientists themselves are quick to point out. Don't take it as a personal insult, merely as an opinion from someone else in the field, but I'd wager you're quite young. When you have a few years of experience under your belt, I'd be willing to bet that you won't be saying "there is nothing mysterious going on in the brain" with quite so much confidence.

We do not yet understand the entirety of how the brain works. It's still very much a work in progress for humanity and there is most certainly a great deal that remains a mystery to us. I'm sure we'll uncover it some day, and we'll be able to make categorical remarks on whether thoughts, memories and knowledge exist or not, and if they do, whether they're immaterial or not. But today is not that day. Currently, your assertions are backed up only by shaky and highly debatable evidence, with plenty of contradictions to boot.

Again, neuroscience is a subject I wouldn't be so quick to trivialise.


Memory is almost certainly in the connections that have formed between neurons. There is more than one type of connection and different types behave differently.
Neurons can connect more than once to other neurons.

There are also more than one type of neuron and they behave differently to the same connection type and make yet other connections and connect in different ways.

Let's do a very simplified, and almost certainly wrong, estimate.

You have 100 neurons in a dish. Each neuron can form (or not form) connections to 99 other neurons. Each neuron can make 10 types of connections.
Each neuron can differentiate between having one to twenty connections of the same type (it can sense how many connections it has).

One neuron has 99 neurons x 10 connection types x 20 connection strengths = 19.8 k connections

Assuming the presence or absence of a connection corresponds to one bit (1 and 0 respectively), this means that a dish of 100 neurons can store 19.8MB of information.

Assuming there are 200 neuron types in the brain (this is pretty darn conservative since a single one of the six layers of neurons in the cortex has above 20 distinct types of neurons, differentiated by cell surface receptors), that means that the same 100 neurons can exist in 200^100 = 1.27 x 10^230 different combinations of neuron types.

Sure, each specific dish combination only stores 19.8 MB, but that same connection pattern means different things if translated to a different set of neuron types.

Now, how many billion neurons do we have in our brains? Wikipedia tells me we have 100 billion...
Simply scaling up the estimate (assuming that each neuron can contact up to 100 others) puts the estimated information capacity of a human brain at 20 petabytes.

(and don't get me started on the glial cells, which outnumber the neurons and do play some sort of role. We haven't pinned it down exactly. )

Yes, that's all very nice, but it's only a hypothesis. You have no evidence that supports that things are like that, only that they could be. And how would the brain turn reality into binary data? Would it spontaneously create and destroy connections between neurons to do so? That's something that could be observable on a microscopic scale. Why hasn't such an occurrence been reported yet? No experiments conducted? And how would it translate all the information in reality into binary? In real life, we gave every pixel of colour a specific code so that we can recreate images. Leaving audio and video aside, what about other sensations? How is heat/cold, pain, smell, taste, touch and all the other senses stored in our memories? How do you transcribe that into a binary code?

Just because something could be, doesn't mean it is.


Take your computer chipboard. Cut a few random conduction paths (that's those silver lines)
Run wires from random conduction paths to other random conduction paths.
Then cut a (small) hole out of the chipboard, taking care to remove at least one small chip.

Does your computer still work? My guess is no.
Does your computer still have electricity running through it? Do all the bits still work the way those bits are supposed to work? Yes, if you turn it on.

Brain dead is the same deal.

Just because its made of the same stuff and individual bits still exist, does not mean that if you take away some or disrupt the connections that the thing works in the same way.

Actually, you're quite wrong on this one. If brain damage has destroyed some connection, then no, electricity is not running the way it should. Just like in a PC, when you cut a conduction path, electricity is no longer circulating through the chipboard the way it should.

So in that case, you could trace the brain death to a physical cause. And as you know, there are cases of brain dead people without a discernible physical cause.

Rogue Shadows
2011-11-26, 06:56 PM
Ok, that does it. I'm aiming to do a PhD in a neuroscience related area and from the roughly 40 research papers I have read on the subject of neurotransmitter receptors quite firmly concludes that there is nothing mysterious going on in the brain.

Neurons interact with each other by more than signal propagation. There is growth signals, signals that stimulate large areas, signals that remodel or remove or add connections, etc.

The interactions between those signals and how those process and store information is the essentially the field of neuroscience.
This is so insanely complex that I doubt we will be done any time before I'm in the ground.

At the same time, the complexity comes from the interactions and emergent phenomena that arise from them. Not from some mystery... thing.


Memory is almost certainly in the connections that have formed between neurons. There is more than one type of connection and different types behave differently.
Neurons can connect more than once to other neurons.

There are also more than one type of neuron and they behave differently to the same connection type and make yet other connections and connect in different ways.

Let's do a very simplified, and almost certainly wrong, estimate.

You have 100 neurons in a dish. Each neuron can form (or not form) connections to 99 other neurons. Each neuron can make 10 types of connections.
Each neuron can differentiate between having one to twenty connections of the same type (it can sense how many connections it has).

One neuron has 99 neurons x 10 connection types x 20 connection strengths = 19.8 k connections

Assuming the presence or absence of a connection corresponds to one bit (1 and 0 respectively), this means that a dish of 100 neurons can store 19.8MB of information.

Assuming there are 200 neuron types in the brain (this is pretty darn conservative since a single one of the six layers of neurons in the cortex has above 20 distinct types of neurons, differentiated by cell surface receptors), that means that the same 100 neurons can exist in 200^100 = 1.27 x 10^230 different combinations of neuron types.

Sure, each specific dish combination only stores 19.8 MB, but that same connection pattern means different things if translated to a different set of neuron types.

Now, how many billion neurons do we have in our brains? Wikipedia tells me we have 100 billion...
Simply scaling up the estimate (assuming that each neuron can contact up to 100 others) puts the estimated information capacity of a human brain at 20 petabytes.

(and don't get me started on the glial cells, which outnumber the neurons and do play some sort of role. We haven't pinned it down exactly. )


Of course, that contains so many simplifying assumptions to be basically pointless. But I hope it was demonstrative of the power of combinatorial problems.

Is it any wonder that we haven't deciphered it yet? No, not at all surprising.


Take your computer chipboard. Cut a few random conduction paths (that's those silver lines)
Run wires from random conduction paths to other random conduction paths.
Then cut a (small) hole out of the chipboard, taking care to remove at least one small chip.

Does your computer still work? My guess is no.
Does your computer still have electricity running through it? Do all the bits still work the way those bits are supposed to work? Yes, if you turn it on.

Brain dead is the same deal.

Just because its made of the same stuff and individual bits still exist, does not mean that if you take away some or disrupt the connections that the thing works in the same way.

I'm given to understand that the "short version," as it were, of this is that the human brain has more ways of interacting with itself than there are atoms in the known universe.

Or maybe it was stars in the known universe. Stars, atoms, either way the sheer complexity of the human brain is, amusingly, beyond its own ability to comprehend.

jseah
2011-11-26, 07:43 PM
Last post today.


I never said, however, that complexity arises from 'some mystery thing.' I said that there is more to neuroscience that mere electricity and biochemistry, as neuroscientists themselves are quick to point out.
And that "more than electricity and biochemistry" part of neuroscience is in the interactions between those individual systems.

I seriously doubt any respected neuroscientist would claim that the brain violates known chemistry and physics.

Sure, my knowledge of neuroscience is more on the biochemistry side so the level of knowledge I have about how neural networks work is maybe 1st year undergraduate.

I have this book on synapses in the brain sitting on my desk right now. I haven't had time to read more than the first chapter, but that covered the history of neuroscience of the synapse.

No funky immaterial stuff. Just biochemistry really.


I'm sure we'll uncover it some day, and we'll be able to make categorical remarks on whether thoughts, memories and knowledge exist or not, and if they do, whether they're immaterial or not. But today is not that day. Currently, your assertions are backed up only by shaky and highly debatable evidence, with plenty of contradictions to boot.
Actually, basically all of neuroscience is trying to reduce the complex brain into understandable terms.

No one I know of is looking at it and saying, "oh hey, I think parts of information processing in the brain involve immaterial stuff"

Everyone is trying to find out what this particular material bit is doing.

Or taking high level behaviour (eg. flying planes (http://neural.bme.ufl.edu/page13/assets/NeuroFlght2.pdf)) and trying to decipher the material bits involved.


No one is looking for immaterial stuff because it's so much a part of our science that everything is material.


Again, neuroscience is a subject I wouldn't be so quick to trivialise.
I freely admit that neuroscience isn't trivial. That's what makes it interesting after all!

The complexity is in the interactions of all its parts. (or in the case of the group I hope to do a PhD with, in the interactions of parts of one specific neurotransmitter receptor)

The complexity is not in how the brain does anything that isn't in physics. Because it doesn't.

We don't know for sure it doesn't. But that's about the same as saying "we don't know for sure that if you let an apple go, it will drop to the ground".
Things don't suddenly stop being physical just because they are complicated.


Yes, that's all very nice, but it's only a hypothesis. You have no evidence that supports that things are like that, only that they could be.
<...>
Just because something could be, doesn't mean it is.
True. I can't show that what I said there is the exact truth.
And almost certainly, I have gotten stuff wrong. My understanding of neuroscience is nowhere near complete.

That was just a demonstration of how the brain might do it without resorting to non-physical explanations.
It was to show that non-physical explanations aren't required for memory storage.

Physical explanations are enough, and without good reason, we assume no non-physical stuff.


So in that case, you could trace the brain death to a physical cause. And as you know, there are cases of brain dead people without a discernible physical cause.
Ok, I'm going to have to ask you provide a source for this. Link me a case of a patient who is/was brain dead and did not suffer some medical complication, accident or was just plain too old.
And the brain still has to be normal. Meaning normal EEGs, normal MRI activity, no funny business with protein aggregates or dead neurons like in Alzheimers.
Every physical cause of brain death has to be ruled out. The brain works, to all our tests, including post-mortem if any, and the patient should be up and around, but somehow isn't. With no known prior cause too.

Mind you, when you say without discernible physical cause, that is what I understand by it.

What did the doctors say about that case?

NichG
2011-11-28, 07:10 AM
So getting back to time travel, the one-billiard case seems to show that you tend to have a multiplicity of possible solutions when you introduce time travel rather than the single solution you had before without time travel. I wonder, is it possible to come up with a case where you can set things up so that there are no solutions if you are allowed to add additional elements to the system. It may be that this only occurs if you allow other infinities. If thats true, you can use self consistent deterministic time travel to temporarily make use of massive amounts of energy (you can't keep it, but you can 'borrow' it to do things that involve zero net work). This is similar to how the time travel solutions temporarily deconserve momentum (by introducing extra moving objects temporarily that resolve later).

Lets consider a system with a time travel wormhole and a roving black hole. Can we create a situation where any trajectory that could save us would be interrupted by falling into the black hole's event horizon (since collisions that occur within the event horizon are effectively totally inelastic and therefore break time reversal symmetry). Similarly, if we do the billiards problem but force the collisions to be at least partially inelastic, I wonder if the results of the paper still hold. Since every collision loses energy, I don't think they can unless you also track the thermal effects, transmitted photons due to thermal radiation, etc. This means that we can break the laws of thermodynamics as to the growth of entropy when self-consistent time travel is involved (because we have a state that randomizes and then de-randomizes). It's also interesting that somehow we must avoid radiating away energy in the form of photons in order to pull off the right sort of loop, which suggests that we'd have to have particles capable of colliding with photons constantly stream from the wormhole mouths to 'deflect' the photons on self-consistent paths. As such, we'd probably see high energy gamma rays (enough to cause pair production) in the vicinity of the wormhole at an intensity comparable to the thermal radiation of the billiard.

If we connect the wormhole mouths to a ship, can we use this to create a spacetime swimmer using the temporary deconservation of momentum? That is to say, a system that displaces itself by a fixed distance every time the time travel cycle occurs, ending with the same momentum and energy but having been displaced. If so, we might have a nice design for a drive that doesn't use up its reaction mass.

Similarly, I wonder if a time travel wormhole in the presence of an ambient ideal gas exerts a net negative or positive pressure due to the occurrence of coincidental collisions involving particles that were traveling towards the inlet.

jseah
2011-11-28, 08:53 AM
XD Those are some issues I hadn't thought about fully.

The thing is, all those temporary non-conservation of momentum, energy, etc. only happen at the wormhole. You can say that its the wormhole that's causing this.

You know, because it's a wormhole. If time travel simply involved turning time direction backwards for a set of particles (meaning it would still exist but travelling backwards in time), what you have is a method of turning matter into anti-matter. At least at first glance.
Except of course, if you draw the Feynman diagram, then this time-reverse matter creation is identical in everyway to a matter-antimatter annihilation.


So, to answer your implied questions, yes. Yes, this does break all manners of conservation laws, at least at the interfaces for the time travel, although not in between.
Conservation of momentum: Wormholes break symmetry wrt. translation
Conservation of energy: Wormholes impose a time direction (you can tell which time direction the wormhole points in, due to consistency principle)


So getting back to time travel, the one-billiard case seems to show that you tend to have a multiplicity of possible solutions when you introduce time travel rather than the single solution you had before without time travel.
And, IIRC, the paper says they could assign quantum probabilities to each of the possible solutions.

If you take the billiard ball + wormhole + table as a quantum system, you can then say this:
"When I observe (decohere) the system, the chances I will get X solution is P"
For any X solution.
Inconsistent Xs will have P = 0.
I expect that some consistent ones will still be 0 due to interference.


It's also interesting that somehow we must avoid radiating away energy in the form of photons in order to pull off the right sort of loop, which suggests that we'd have to have particles capable of colliding with photons constantly stream from the wormhole mouths to 'deflect' the photons on self-consistent paths.
Not quite true.

Let's say the billiard ball is in thermal equilibrium with the table. Let's say the only thermal contact they have is through radiation.

This means that photons that are hitting the ball contribute equal amounts of energy on a macroscale to the photons that are leaving the ball.

Now, let's further assume we can track the worldline of one specific packet of energy that causes a flourescent reaction. (for simplicity)
It starts in the room and hits the present ball rolling towards the wormhole. This causes an electron to jump up an energy level. The ball rolls into the wormhole and exits in the past, carrying that exicted electron (and its packet of energy we're tracking) with it.
The electron now comes down from its excited state and the photon leaves. In the "past".

So, the photon from the future is released in the past. It now continues along its merry way and says hi to its (subjective) past self and proceeds to watch its past self disappear into the wormhole to complete the loop.

Subdivide the energy packets into Planck energies and you will see that creating consistent loops on the billiard ball case is trivial.
Alot of photons from the past and future go into the wormhole and come out. Some of them will be doing this more than once. There is no inconsistency.


FYI, this is the same logic behind the one-atom key. Its a key made of exactly one atom passed in a million billion billion loops. All of which are mediated by the exact same time travel event.
You're just bringing something that has time travelled once, again. And again, and again... etc.
If you have more than one atom in your key, then you have a more complex subdivision of loops. Maybe hidden in there is an atom that has a closed loop (aka. "borrowed").

NichG
2011-11-28, 05:57 PM
XD Those are some issues I hadn't thought about fully.

The thing is, all those temporary non-conservation of momentum, energy, etc. only happen at the wormhole. You can say that its the wormhole that's causing this.


Thats why you want to take that wormhole and harness it to a device to get use out of it. Basically, imagine a device that has a part of itself threaded through the wormhole. Because of temporary deconservation you'll end up with a wormhole 1ft to the left (or 1 um to the left) after the billiard came and went, and so the device will also move. If you keep cycling the same billiard through the wormhole you can end up with a ship that can travel through space without consuming energy or reaction mass, so long as it does not move up any potential gradients. Thats a pretty useful thing.



You know, because it's a wormhole. If time travel simply involved turning time direction backwards for a set of particles (meaning it would still exist but travelling backwards in time), what you have is a method of turning matter into anti-matter. At least at first glance.
Except of course, if you draw the Feynman diagram, then this time-reverse matter creation is identical in everyway to a matter-antimatter annihilation.


I like the image of a time travel scheme being that you focus a lot of light on a spot, a person appears along with their antimatter double, they go and do their thing, then at the appointed time a person willingly steps into their antimatter version and annihilates ('effectively' being transported into the past). Of course, thermodynamically this basically means you never get the 'go' for time travel since you need to overcome the improbability that the massive energy you pump in does nothing rather than forming the matter/antimatter copy of the time traveler.



So, to answer your implied questions, yes. Yes, this does break all manners of conservation laws, at least at the interfaces for the time travel, although not in between.
Conservation of momentum: Wormholes break symmetry wrt. translation
Conservation of energy: Wormholes impose a time direction (you can tell which time direction the wormhole points in, due to consistency principle)


Well I'd like to imagine the wormhole as part of physics as well. There are two ways it could effectively break conservation. Way 1 is that it is a big reservoir (huge mass, energy) and so when conservation is apparently broken its just because energy and momentum are being transferred from the wormhole. This 'breaks' conservation the same way the earth breaks conservation, in that if you properly account for the wormhole then you're exactly conserved at each moment in time.

The way I'd prefer to see it though uses the idea from relativity that simultaneity is actually a non-trivial thing. That is to say, the existence of the wormhole makes it very clear you can't simply cut time into slices and ask 'is the total energy/momentum of the universe the same in this slice as all other slices?' because a slice is not a well-defined object in a universe with wormholes. That is to say, we've replaced time translational invariance and space translational invariance with a new translational invariance that combines both time and space.



And, IIRC, the paper says they could assign quantum probabilities to each of the possible solutions.

If you take the billiard ball + wormhole + table as a quantum system, you can then say this:
"When I observe (decohere) the system, the chances I will get X solution is P"
For any X solution.
Inconsistent Xs will have P = 0.
I expect that some consistent ones will still be 0 due to interference.


It reminds me a lot of Feynman diagrams with loops, where there's an infinite number of more complex loops you could propose but their multiplicity grows more slowy than their relative probability decreases, so you get a finite result. Of course, certain systems have integrals here that diverge and require weird divide by infinity tricks (renormalization), and in quantum gravity you can find cases that are simply not renormalizable.



Subdivide the energy packets into Planck energies and you will see that creating consistent loops on the billiard ball case is trivial.
Alot of photons from the past and future go into the wormhole and come out. Some of them will be doing this more than once. There is no inconsistency.


I'm more concerned with things that are thermodynamically irreversible occurring as part of a loop. The likely answer is that the self-consistency requirement (hard requirement) lets you break the 'soft' requirements of thermodynamics (which only apply statistically, not exactly). That is to say, you can have cases where something irreversible reverses due to very precise happenstance.

jseah
2011-12-01, 08:21 PM
I won't reply to all of them. Suffice it to say that most of what you are talking about is possible.

The thing about simultaneity and conservation laws is that on first glance, consistent time travel appears to "conserve" the number of unique objects and energy that are not time-clones of themselves.

However, it is possible to create a consistent loop that involves an increase in the number of unique objects or energy for the length of the loop.


I'm more concerned with things that are thermodynamically irreversible occurring as part of a loop. The likely answer is that the self-consistency requirement (hard requirement) lets you break the 'soft' requirements of thermodynamics (which only apply statistically, not exactly). That is to say, you can have cases where something irreversible reverses due to very precise happenstance.
Like most of thermodynamics, this is a statistical thing. Since there will be a very large number (probably infinite) of consistent loops that are but microscopic variations of each other, there will be vastly more loops that increase entropy than those that do not.

Much in the same way that there are vastly more ways to distribute the air in a room over the whole volume compared to the ways to pack it in a corner.

However, if you could somehow link a decrease in entropy to the consistency requirement, then you could do it. Or simply arrange things such that after the probabilities of the consistent loops are accounted for, the net result is that the average entropy balance of the sum of all loops is negative.
Then yes.

I expect this to be an extremely unlikely occurence unless deliberately engineered, and yes, it does break the second law of thermodynamics. You do have a time travel wormhole after all.

NichG
2011-12-01, 11:12 PM
So amusingly, I'm actually in a campaign with strong time travel themes right now. It doesn't quite use the self-consistent model, but instead uses a 'second time dimension' model. The trick is, certain creatures, PCs, and time travellers are 'higher dimensional creatures' who treat time 1 as a spatial dimension to 'some degree or other' (this is all very vague since we don't quite know yet).
I've been trying to figure out ways to safely experiment on the nature of Time-2 without erasing myself from existence :smallsmile:

kieza
2011-12-02, 01:54 AM
Continuum's causality works like this:

Time-travellers can cause changes to the timestream. However, it has to be done in such a way that nobody notices: the final outcome has to be exactly the same, as far as any non-time-traveller and the past selves of the parties involved can tell. There are no paradoxes, because every "legit" time-traveler out there is simultaneously working to fix them as they arise. Whenever something paradoxical occurs, the time-travel tech informs appropriate parties (usually those closest to the paradox) by way of a nasty physical sensation. They then need to go and do something about the paradox. If they fail, people further up the ladder start getting fragged (that's the technical term), and they then get involved in patching the timeline. If it gets bad enough, the Inheritors (Sufficiently Advanced Aliens) show up and repair the frag with overwhelming force, up to abducting the affected parties and replacing them with stand-ins for the smooth functioning of the timeline.

So, you can kill Hitler, but you or someone else will then have to go back and impersonate Hitler until his eventual death and leave his corpse there, or grow a clone and give it Hitler's memories, or administer advanced first-aid after you've left Hitler for dead, then wipe Hitler's memories of having died.

Example (I think this one's out of the manual):
Adam and Bob are novice time-travellers with a shared apartment. Adam gets home at 5, grabs the last beer out of the fridge, and sits down to watch the game. Bob gets home at 5:15, goes to the fridge, and finds no beer. There are five beers in the pantry, but they're nasty and warm. So, Bob spans back an hour (4:15) looking for a cold beer in the fridge. He finds one, spans forward (5:15) and heads out to watch the game. Adam is doubled over from frag. "Oh crap," says Bob, "is this your beer too?" Adam swears at Bob to fix the frag, so Bob thinks quickly, goes back in time two hours (3:15), finds a full six-pack in the pantry, and puts one in the fridge (leaving five in the pantry). He then spans forward an hour (4:15), grabs the now-cold beer, and spans forward half an hour (4:45) and places it in the fridge so that Andy had something to take out when he got home at 5. He then spans forward to 5:15, just after he found Andy, helps him off the floor, and they both sit down to watch the game.

Now, from Andy and Bob's POV, when Andy gets fragged, it's because they both have the same beer. But because they're time-travelers, they know that's impossible. So it's up to Bob to find some way for what they just did to not violate causality, which he does by planting another beer when nobody's looking. They never had the same beer, it just looked like they did until Bob figured out the trick. Since the beers are, for all intents and purposes, identical,

Another, nastier example:
Charlie is a narcissist (a time-traveller who tries to ignore causality), and he has a beef with Adam and Bob's future selves. He comes back and frags Adam by stealing Adam's newspaper before he bought it. When the frag hits Adam, he and Bob can't figure out what happened, because neither of them caused the frag. So, they head to the central archives on Atlantis, where Bob's future self has left Adam a message explaining who Charlie is and where they can find him. Unfortunately, when Adam and Bob show up armed for bear, they accidentally kill Charlie's past self, who doesn't know who they are and hasn't yet fragged Adam. Now Adam is doubly fragged, because he has prevented Charlie from fragging him in the first place. A couple more attempts to fix the problem also go awry, until Adam can hardly move. Finally, the Inheritors show up and remove Adam's body. An Inheritor impersonates Charlie from the time he was killed onwards, including fragging Adam in the first place. They also replace the newspaper and everything else that was used in the misadventure and prevail upon Bob to leave the note for Adam.

Again, there's no paradox. When future-Charlie frags Adam, he changes the timeline. When Adam accidentally frags himself by killing past-Charlie, he changes it even more. But the Inheritors eventually put it back so that the end result is the same as before the changes were made.

jseah
2011-12-03, 07:26 AM
NichG:
Treating time as a spatial dimension is pretty screwy, since if you go backwards in time, that implies that you have to pass through all the intervening bits of time as well.

That just gets even more screwy that direct time teleportation.

kieza & continuum:
This is part of the reason why I dislike Continuum's model. It disallows the more interesting (to me) portions of time travel involving extremely complicated loops.

You can't pull out an ontological paradox in Continuum, much less engineer a true bootstrap out of nowhere.

--------------------------------------------------------------------------------

Anyone have anything more complicated than ontological loops? I can't think of anything. =(

NichG
2011-12-03, 08:56 AM
NichG:
Treating time as a spatial dimension is pretty screwy, since if you go backwards in time, that implies that you have to pass through all the intervening bits of time as well.

That just gets even more screwy that direct time teleportation.


Well, its more that there are two temporal dimensions. I'm thinking of it as a sort of 'helix time travel' model, but I don't have in-game confirmation of my suspicions yet (doing the experiment next game, we'll see if I edit myself out of existence).

The idea is, if you return to the same spot in Time-1, you're actually there at a later point in Time-2 (in fact, such recurrences would be the only way to guarantee you're actually moving in Time-2 at all). I think of Time-2 as essentially determining direction of causality. An event earlier in Time-2 that includes time travel can only influence events that are later in Time-2. So if you go back and kill your own grandfather, you're killing him at a later point in Time-2 than determined your own birth, and so there is no causal connection between that action and whether or not the you that time traveled back actually exists.

Now posit certain beings that naturally move forward in Time-2 as part of their normal existence (say, beings that use time travel events in their own biology). I'm trying to figure out what the consequences of that are observationally.

The experiment will be:

Have a whiteboard in a dark room that has been left totally unobserved for an hour. Go into the room and have the lights turn on exactly synchronously with a time traveler being sent one hour into the past with instructions to go to the whiteboard and write on it. The dark room thing is to try to avoid all possibility of causal connection between what is observed to be written on the board and what the time traveler will write on the board.

Potential outcomes:
1. The total text that was going to be written is found on the whiteboard when the lights go on (self-consistent model). The time traveler is waiting somewhere and does not 'pop out of existence'.
2. No text is observed, and the time traveler is never seen again (suggests all entities involved are strictly Time-1 beings and/or a 'branching' time travel model is in effect).
3. Text appears at some rate on the whiteboard with a delay following the time travel event (Back to the Future model, possible outcome for a Time-2 integrated observer).

The interesting thing would be in case 3 if different observers saw different things based on whether they were Time-2 beings or not. That is to say, two people looking at the whiteboard might see different outcomes if they're moving through Time-2 at different rates relative to their motion in Time-1. This sounds kind of silly, but it's sort of like simultaneity effects in relativity with observers in different reference frames. You'd be able to measure someone's 'temporal velocity' by how long it took them to see the writing appear, and how fast it appeared relative to the actual rate of writing.

jseah
2011-12-03, 05:32 PM
Just had an interesting discussion with a friend in RL over lunch.

Turns out that two time travellers fighting each other in consistent time travel is... not conceptually easy.
Each time traveller can send information from the future and can set up ontological loops.

This kind of combat is "fought", not in the real world, but in the all the loops the time travellers could have launched. All of the possible loops, at the same time, each time traveller fighting to negate their counterpart's time loops.
We eventually agreed that most of these loops would negate each other by resulting in unresolvable inconsistencies (paradox), and so the final loop that actually happens comes from the remaining consistent loops. Which may or may not include the execution of various possible ontological paradoxes despite those being immediately possible (but generate inconsistencies later).

Eg.Time traveller B has a sniper rifle and a landmine. He wants to kill time traveller A.

We will restrict both of them to one time jump to keep things simple.

What happens: B finds A on the street and shoots him in the head. A dies, the end.

Explanation:
Time traveller A could have shot time traveller B with B's sniper rifle and then complete the ontological loop by telling himself. Cost one time jump.
However, B can stop A's ontological loop by using his one time jump to plant the landmine whereever A will try to snipe him from, creating his own ontological loop.
This prevents A's loop from completing and thus A's ontological loop of shooting B will generate an inconsistency. (he couldn't have gotten a sniper rifle without killing B then time jumping, which he won't do if his future self gets blown up and never gets to communicate where B is)

Thus, B's potential ontological loop has stopped A's potential ontological loop. So at the end of the "fight", neither loop happens and B's one time jump and landmine aren't used...
Of course, other loops and other explanations could be found.

This presents a problem. As a story writer, you have very little way to reasonably explaining what went on. The final result just looked like someone shot someone else, neither of whom used their time travel power.

As a game though... that's where this gets awesome. More awesome than Continuum, I mean.

A GM can present a starting scenario, all relevant information to which will be recorded.
When the players are using their time travel, they propose what they want to do to the GM, whose villians may counter with further time loops. If a consistent loop can happen and a "break" occurs where both parties can't or won't enact further loops, this potential future state can then be 'recorded' so that the game can rewind to that point if further future states turn out to be unresolvable.

This allows you to play the same fight multiple ways, exploring possible options before negating those you don't like (a time traveller can very easily cause an inconsistency by time travelling to a point where he knew he did not time travel to, this would lead to a rewind back to the last recorded state)

As a method of creating ontological loops, players can invoke them by saying "my character will be time travelling back to this point in time, with so-and-so equipment". Details about the equipment can be left hazy until used to make completing the loop easier (eg. Character appears with a tommy gun and an indeterminate number of rounds of ammunition. Later, the past self grabs a past tommy gun and at least the number of rounds that the future self fired to complete the loop)

Furthermore, as information becomes available, players can invoke ontological loops by saying that future selves would have informed their character of that information and lay out a method of confirming that information.
If the history of past actions can be recalled, subsequent modifications can be simple tweaking. But more complex future information schemes might have the game rewind to the last 'recorded' state and play out from there, invoking the ontological loop carrying the future information as per normal.

Paseo H
2011-12-04, 12:35 PM
I'm still working out how to actually make it work, but my designated Godmode Sue/All Powerful Bystander/Time Lord is one of these types, can move semi-freely through time/space, has limited omniscience, etc.

The thing is, you'd think this would make her dangerous, but the fact of the matter is, now that she has a greater knowledge of the effects of any actions she takes, that makes her far more fettered than unfettered.

So what she's doing now, primarily, is empowering others to act instead of taking too many direct actions of her own.

She is basically a Humanoid Abomination of sorts, her heart was replaced with the heart of a low grade Eldritch Abomination, a beneficial one however. Think Dr. Manhattan or a Time Lord. Through the power of that being's blood that now flows through her veins, she can essentially enact a Touched By Vorlons on people.

NichG
2011-12-05, 03:42 AM
The game with the experiment was today. Outcome was #4, random gobbledegook appears on the whiteboard the instant the lights turn on and the robot sent back in time has been modified with nonsense tech.

Conclusion: Experiment was pranked by a marauder.

jseah
2011-12-05, 06:30 PM
It appears to me that in time traveller vs time traveller conflicts, the person with the best understanding of time travel will be able to execute more complicated loops and thus be able to win.
Equipment advantages are secondary compared to the sheer insane power of consistent time travel, although they do help. Friends who can't time travel are probably not much more useful than having a gun. The same applies to critical pieces of information (about as useful as a gun)

And of course, such a conflict is not decided by what actually happens, but what loops could happen (and what could negate those loops).
Having an advantage in time travel concepts, like being able to note primary causes and estimate what future effect changing things would have, could result in a victory where NONE of the concepts are even used at all.

Simply knowing them, when your opponent does not, could be enough to negate all those loops and have you win. Without any indication as to what allowed you to win in the first place.


How and why one side of the conflict wins may not be apparent, and might never be possible to find out in-universe. Even with time travel to help.


Its quite... confusing. Simply having something or knowing something could "affect" the future by changing which time loops are consistent; without actually having a direct causal effect on the universe. (apart from minor butterfly-effect variations)

Eg. having a gun in your jacket that no one notices could make you win a time-travelling fight. Even though the final resulting time loop never has you draw the gun, much less use it.
And afterwards, when you and a time clone punch out the other guy, it isn't at all obvious that the gun you have in your jacket helped you win. HOW it helped is even harder to understand.

Its almost as if there's a kind of meta-causality, represented by which objects and events allow which time loops (if X object was not here, then Y time loop would be consistent). And while meta-causality determines the actual loops that happen, objects and events that have significant meta-causality effects may not necessarily contribute significantly to the in-universe causes of the actual loops that do happen.

NichG
2011-12-05, 07:38 PM
And of course, such a conflict is not decided by what actually happens, but what loops could happen (and what could negate those loops).
Having an advantage in time travel concepts, like being able to note primary causes and estimate what future effect changing things would have, could result in a victory where NONE of the concepts are even used at all.


This is a bit of an aside, but a lot of strategic board games play out like this. One in particular that is very heavy on this kind of thing is Go. There are a lot of local games that play out until the penultimate move (because there's rarely any profit in actually completing the local game, compared to moving elsewhere). What this means is that there are all these places where if one player got an extra move somehow (like if the other player were forced to move away), the fate of those stones could be reversed. In the early game, those spots aren't valuable compared to far away moves, but all the value in the late game lies in those places.

So a lot of the late game play revolves around taking advantage of the fact that you could do something and capture pieces, even if you never do that particular thing.

jseah
2011-12-06, 04:36 PM
Building on the idea of meta-causality.

As an author of a consistent time travel story, I know or can introduce any element of a scenario. However, when considering what time travel solution to write, I can isolate possible loops involving various elements and decide what I want to happen.

Since I obviously don't plan all elements of a situation before I start, it would be nice to be able to tell, even as a rough rule of thumb, what sort of meta-causality effect any particular additional element I choose introduce has.

Eg.
There are three robbers planning to hold up a jewelry shop. Our time travelling hero of a 14 year old girl Anne has the trust of the police (they will listen to her tip-offs) but she will not be satisfied if there are any casualties. (ie. will time travel and cause inconsistency) EDIT: at this point, Anne doesn't know its a consistent time travel, and she's a bit too young to actually understand exactly how it works.

The agreed arrangement is that the police will send her a text message about a crime, the location and time it occurred, as well as its outcome, after the incident has been resolved. She will then time travel to half an hour before the time of the crime and forward the message to the police contact number she has been given.

By herself, Anne has no way to know that any specific crime occurred.

The null (no time travel) solution is inconsistent. The police will receive a call for help as the robbers hold hostages and they will send Anne a message, which causes her to time travel.
If the police raid the store just as the robbers come in (standard procedure for forewarning messages), one of the robbers will panic and fire his gun. This kills a salesgirl and thus the basic loop of (Anne gets message, Anne travels back in time, Anne sends message) is inconsistent as Anne's message of a casualty will not result in a direct police raid.

So, the solution involves some higher-order time loops.
Perhaps with a future Anne telling the current Anne the message "robbery, jewelry store, 11.45am, all three criminals detained, no casualties" and then insisting that past Anne go with future Anne to the crime scene.
Perhaps it involves a time travelling gun.

But lets say a particular independent loop (it can happen at any similar situation) can result in Anne borrowing a gun from the police. That loop wouldn't normally occur since she never thinks of wanting a gun (and thus the null solution of no-gun is consistent).
- I did not pre-plan the existence of this acquire-a-gun loop before writing this. I am considering the existence of this loop as an element an author can introduce on the fly to guide the final consistent solution where he wants it to go.
But now, the fact that the null and basic solution to the robbery are both inconsistent could pull in other loops like the acquire-a-gun-loop, whose successful execution might result in a future-Anne-with-a-gun resolving the robbery incident with no casualties.

Is there an easy way to tell how Anne having a gun might affect the situation? I suspect not. I can't even tell what a 14 year old girl is going to do with a gun she can't aim with. Give it to someone else?
What if I make one of the robbers carry a bomb with a deadman's switch? Its hard to decide what could happen.